Physics - JEE Main 2021 July Chapter-Wise - MathonGo

Download as pdf or txt
Download as pdf or txt
You are on page 1of 203

Alternating

  Current JEE Main 2021 (July) Chapter-wise Questions


Questions with Answer Keys MathonGo

Q1 (20 July 2021 Shift 1)

AC voltage V (t) = 20 sin Ωt of frequency 50 Hz is applied to a parallel plate capacitor. The separation
between the plates is 2 mm and the area is 1 m . The amplitude of the oscillating displacement current for the
2

applied AC voltage is ___ [ Take ε0 = 8.85 × 10


−12
 F/m]

(1) 21.14μA

(2) 83.37μA
(3) 27.79μA

(4) 55.58μA

Q2 (20 July 2021 Shift 1)

In an LCR series circuit, an inductor 30mH and a resistor 1Ω are connected to an AC source of angular
frequency 300rad/s. The value of capacitance for which, the current leads the voltage by 45 is .
∘ 1 −3
× 10  F
x

Then the value of x is ___

Q3 (20 July 2021 Shift 2)

For a series LCR circuit with R = 100Ω,

L = 0.5mH and C = 0.1pF connected across

220 V − 50 Hz AC supply, the phase angle between

current and supplied voltage and the nature of the

circuit is :

(1) 0 , resistive circuit


(2) ≈ 90 , predominantly inductive circuit


(3) 0 , resonance circuit


(4) ≈ 90 , predominantly capacitive circuit


Q4 (20 July 2021 Shift 2)

#MathBoleTohMathonGo
www.mathongo.com
Alternating
  Current JEE Main 2021 (July) Chapter-wise Questions
Questions with Answer Keys MathonGo

A series LCR circuit of R = 5Ω, L = 20mH and

C = 0.5μF is connected across an AC supply of 250 V, having variable frequency. The power

dissipated at resonance condition is ×10 2


W

Q5 (22 July 2021 Shift 1)

In a circuit consisting of a capacitance and a generator with alternating emf E g = Eg


0
sin ωt, VC

and I are the voltage and current. Correct phasor diagram for such circuit is:

(1)

(2)

#MathBoleTohMathonGo
www.mathongo.com
Alternating
  Current JEE Main 2021 (July) Chapter-wise Questions
Questions with Answer Keys MathonGo

(3)

(4)

Q6 (22 July 2021 Shift 1)

Match List-I with List-II :

List − I List − I I

1
(a) ωL >
ωC
(i) Current is in phase with emf
1
(b) ωL =
ωC
(ii)
Current relags behind the applied emf
Maximum current occurs
1
(c) ωL < (iii)
ωC

(d) Resonant frequency (iv) Current leads the emf

Choose the correct answer from the options given below:

(1) (a) − (ii); (b) − (i); (c) − (iv); (d) − (iii)

(2) (a) − (ii); (b) − (i); (c) − (iii); (d) − (iv)

(3) (a) − (iii); (b) − (i); (c) − (iv); (d) − (ii)

(4) (a) -(iv); (b) - (iii); (c) - (ii); (d) - (i)

Q7 (25 July 2021 Shift 1)

#MathBoleTohMathonGo
www.mathongo.com
Alternating
  Current JEE Main 2021 (July) Chapter-wise Questions
Questions with Answer Keys MathonGo

An inductor of 10mH is connected to a 20 V

battery through a resistor of 10kΩ and a switch

After a long time, when maximum current is set up

in the circuit, the current is switched off. The

current in the circuit after 1μs is . Then x

x
 mA
100

is equal to ( Take e
−1
= 0.37)

Q8 (25 July 2021 Shift 2)

A 10Ω resistance is connected across 220 V − 50 Hz AC supply. The time taken by the current to change
from its maximum value to the rms value is:

(1) 2.5 ms

(2) 1.5 ms
(3) 3.0 ms

(4) 4.5 ms

Q9 (25 July 2021 Shift 2)

Two circuits are shown in the figure (a) \& (b). At a frequency of rad/s the average power dissipated in one

cycle will be same in both the circuits.

Q10 (27 July 2021 Shift 1)

A 0.07H inductor and a 12Ω resistor are connected in series to a 220 V, 50 Hz ac source. The approximate

current in the circuit and the phase angle between current and source voltage are respectively. [Take π as
22
]
7

(1) 8.8 A and tan −1


(
11

6
)

  #MathBoleTohMathonGo
www.mathongo.com
Alternating
  Current JEE Main 2021 (July) Chapter-wise Questions
Questions with Answer Keys MathonGo

(2) 88 A and tan −1 11


( )
6

(3) 0.88 A and tan −1 11


( )
6

(4) 8.8 A and tan −1 6


( )
11

Q11 (27 July 2021 Shift 2)

A 100Ω resistance, a 0.1μF capacitor and an inductor are connected in series across a 250 V supply at

variable frequency. Calculate the value of inductance of inductor at which resonance will occur. Given that the

resonant frequency is 60 Hz.

(1) 0.70H
(2) 70.3mH

(3) 7.03 × 10 −5
H

(4) 70.3H

  #MathBoleTohMathonGo
www.mathongo.com
Alternating
  Current JEE Main 2021 (July) Chapter-wise Questions
Questions with Answer Keys MathonGo

Answer Key

Q1 (3) Q2 (3) Q3 (4) Q4 (125)

Q5 (3) Q6 (1) Q7 (74) Q8 (1)

Q9 (500) Q10 (1) Q11 (4)

#MathBoleTohMathonGo
www.mathongo.com
Alternating
  Current JEE Main 2021 (July) Chapter-wise Questions
Hints and Solutions MathonGo

Q1

From the given information,

∈0  A ϵ0 ×1
C =
 d
= −3
 F

2×10
−3 −3

∴ XC =
ωC
1
=
2×10

2×50π×ϵ0
=
2×10

25×4π∈0
Ω

−3

∴ XC =
2×10

25
× 9 × 10
9
=
18

25
× 10 Ω
6

V0
∴ i0 =
XC
=
20×25

18
× 10
−6
 A = 27.47μA

The value of amplitude of displacement current will be same as value of amplitude of conventional current.

Hence option 3.

Q2
xC −xL
tan ϕ =
R

xC −xL
tan 45 =
R

xC − xL = R

ωC
1
− ωL = R

1
− 300 × 0.03 = 1
ωC

1
= 10
ωC

C =
10ω
1
=
10×300
1

C =
1

3
× 10
−3

X = 3

Q3

R = 100Ω

XL = ωL = 50π × 10
−3

11

1 10
XC = =
ωC 100π

  #MathBoleTohMathonGo
www.mathongo.com
Alternating
  Current JEE Main 2021 (July) Chapter-wise Questions
Hints and Solutions MathonGo

XC ≫ XL

& |XC − XL | >> R

Q4

XL = XC (due to resonance)

so i V V
Z = R ms = =
Z R

2
V 250×250 2
= = 125 × 10  W
R 5

Q5

In capacitor, current lead voltage by π

Q6

(a) For x L > xC , voltage leads the current

(ii)

(b) For x L = xC

(i)

(c) For x L < xC , current leads the voltage

(iv)

(d) For resonant frequency x L = xC , current is maximum

(iii)

Q7
V 20 V
Imax = = = 2 mA
R 10KΩ

For LR - decay circuit I = I max e


−Rr h.

3 +
−10×10 ×x∣0

I = 2mAe

I = 2 m Ae
−1

I = 2 × 0.37 mA

#MathBoleTohMathonGo
www.mathongo.com
Alternating
  Current JEE Main 2021 (July) Chapter-wise Questions
Hints and Solutions MathonGo

I =
100
74
 mA

x = 74

Q8

10Ω

V = 220 V/50 Hz

⇒ i = i0 sin ωt

When i = i

i0 = i0 sin ωt1 ⇒ ωt1 =


π

i0
When i =

√2

i0

√2
= i0 sin ωt2 ⇒ ωt2 =
π

Time taken by current from maximum value to rms value

⇒ (t1 − t2 ) =

π


π
=
π


=
π

4×2πf

=
1

8×50

1
= sec
400

= 2.5 ms

Q9

For figure (a)

2
vras
Pmg =
R

2 2
vrms vms

2
× R =
R
× 1

R
2
= Z
2

2 2
25 = (√(xc − xL ) + 5 )

2
25 = (xc − xL ) + 25

1
xc = xL ⇒ = ωL
ωC

  #MathBoleTohMathonGo
www.mathongo.com
Alternating
  Current JEE Main 2021 (July) Chapter-wise Questions
Hints and Solutions MathonGo
6

2 1 10
ω = =
LC 0.1×40

ω = 500

Q10
XL
−1
ϕ = tan ( ) XL = ωL
R

22
XL = 2 × × 50 × 0.07 = 22Ω
7

−1 22
ϕ = tan ( ) R = 12Ω
12

−1 11
ϕ = tan ( )
6

Z = √X
2
L
+ R
2
= 25.059

V 220
I = = = 8.77 A
Z 25.059

Q11

C = 0.1μF = 10 F

Resonant frequency = 60 Hz ω 0 =


1

√LC

2πfo =
1
⇒ L =
1

2 2

√LC 4π f C
o

by putting values L = 70.3 Hz.

#MathBoleTohMathonGo
www.mathongo.com
Atomic
  Physics JEE Main 2021 (July) Chapter-wise Questions
Questions with Answer Keys MathonGo

Q1 (20 July 2021 Shift 1)

The radiation corresponding to 3 → 2 transition of a hydrogen atom falls on a gold surface to generate

photoelectrons. These electrons are passed through a magnetic field of 5 × 10 −4


 T . Assume that the radius of

the largest circular path followed by these electrons is 7 mm, the work function of the metal is:

(Mass of electron = 9.1 × 10 −31


 kg )

(1) 1.36eV

(2) 1.88eV
(3) 0.16eV

(4) 0.82eV

Q2 (27 July 2021 Shift 1)

In Bohr's atomic model, the electron is assumed to revolve in a circular orbit of radius 0.5Ã …. If the speed of

electron is 2.2 × 16 6
 m/s , then the current associated with the electron will be ___ ×10 −2
 mA. [ Take π as
22
]
7

Q3 (27 July 2021 Shift 2)

The K X-ray of molybdenum has wavelength

0.071 nm. If the energy of a molybdenum atoms

with a K electron knocked out is 27.5keV, the energy of this atom when an L electron is knocked

out will be keV. (Round off to the nearest

integer)

−15 8 −1
[h = 4.14 × 10 eVs, c = 3 × 10  ms ]

  #MathBoleTohMathonGo
www.mathongo.com
Atomic
  Physics JEE Main 2021 (July) Chapter-wise Questions
Questions with Answer Keys MathonGo

Answer Key

Q1 (4) Q2 (112) Q3 (10)

#MathBoleTohMathonGo
www.mathongo.com
Atomic
  Physics JEE Main 2021 (July) Chapter-wise Questions
Hints and Solutions MathonGo

Q1

3 → 2 ⇒ 1.89eV

5 × 10
−4
 T r = 7 mm

r =
mv

qB
⇒ mv = qrB

2 2
(qRB)
⇒ E =
2 m
P
=
2 m

2
−19 −3 −4
(1.6×10 ×7×10 ×5×10 )

= −31

2×9.1×10  Joule 
−52

3136×10
= −31 −19
eV
18.2×10 ×1.6×10

= 1.077eV

We know work function = energy incident − (KE) electron 


ϕ = 1.89 − 1.077 = 0.813eV

Q2

e eω eV
I = = =
T 2π 2πr

−19 6

I =
1.6×10 ×2.2×10 ×7

−10

2×22×0.5×10

= 1.12 mA

−2
112 × 10  mA

Q3

E ka = E k − E L

λk
hc
= Ek − EL

hc
EL = Ek −
λk
a

−7

= 27.5KeV −
12.42×10

−9
eVm

0.071×10  m

EL = (27.5 − 17.5)keV

= 10keV

  #MathBoleTohMathonGo
www.mathongo.com
Capacitance
  JEE Main 2021 (July) Chapter-wise Questions
Questions with Answer Keys MathonGo

Q1 (25 July 2021 Shift 1)

A parallel plate capacitor with plate area 'A' and distance of separation 'd' is filled with a dielectric. What is the

capacity of the capacitor when permittivity of the dielectric varies as :

ε(x) = ε0 + kx , for (0 < x ≤ d

2
)

ε(x) = ε0 + k(d − x) , for ( d

2
≤ x ≤ d)

2/kA

(1) (ε 0
+
kd

2
)

(2) kA


0
+kd
2 ln( )
2ε0

(3) 0
2ε0
(4) kA

2
ln(
2ε0 −kd
)

Q2 (25 July 2021 Shift 2)

If q is the free charge on the capacitor plates and q is the bound charge on the dielectric slab of dielectric
f b

constant k placed between the capacitor plates, then bound charge q can be expressed as : b

(1) q b
= qf (1 −
1
)
√k

(2) q
1

b
= qf (1 − )
k

(3) q b
= qf (1 +
√k
1
)

(4) q b
= qf (1 +
1

k
)

Q3 (27 July 2021 Shift 1)

A capacitor of capacitance C = 1μF is suddenly connected to a battery of 100 volt through a resistance
R = 100Ω . The time taken for the capacitor to be charged to get 50 V is :

[ Take ln 2 = 0.69]

  #MathBoleTohMathonGo
www.mathongo.com
Capacitance
  JEE Main 2021 (July) Chapter-wise Questions
Questions with Answer Keys MathonGo

(1) 1.44 × 10 −4
 s

(2) 3.33 × 10 −4
 s

(3) 0.69 × 10 −4
 s

(4) 0.30 × 10 −4
 s

Q4 (27 July 2021 Shift 1)

In the reported figure, a capacitor is formed by placing a compound dielectric between the plates of parallel
plate capacitor. The expression for the capacity of the said capacitor will be :

(Given area of plate = A )

∣ C1 C2 C3 ∣


 K 3 K 5 K ∣

∣ ∣
∣ ← d → < 2 d → k3 d → ∣

15 Kε0  A
(1) 34  d

15 Kε0  A
(2) 6  d

25 Kε0  A
(3) 6  d

9 Kε0  A
(4) 6  d

Q5 (27 July 2021 Shift 1)

Two capacitors of capacities 2C and C are joined in parallel and charged up to potential V. The battery is

removed and the capacitor of capacity C is filled completely with a medium of dielectric constant K. The
potential difference across the capacitors will now be :
V
(1) K+2

V
(2) K

3 V
(3)  K+2

3 V
(4)  K

  #MathBoleTohMathonGo
www.mathongo.com
Capacitance
  JEE Main 2021 (July) Chapter-wise Questions
Questions with Answer Keys MathonGo

Answer Key

Q1 (2) Q2 (2) Q3 (3) Q4 (1)

Q5 (3)

#MathBoleTohMathonGo
www.mathongo.com
Capacitance
  JEE Main 2021 (July) Chapter-wise Questions
Hints and Solutions MathonGo

Q1

Taking an element of width dx at a distance x(x < d/2) from left plate

(ε0 +kx)A
dc =
dx

Δ/2 dn
Capacitance of half of the capacitor 1

C
= ∫
0
1

dc
=
1

 A

0
dx

ε0 +kx

ε0 +kd/2

C
1
=
1

kA
ln(
ε0
)

Capacitance of second half will be same C eq =


C

2
=
kA

2ε0 +kd
2 ln( )

0

Q2

→ → E0
When a dielectric is inserted in a capacitor Due to free charge E = E 0
only After dielectric E ′
=
k

1
qB = qf (1 − )
k

Q3
t

V = V0 (1 − e

RC
)

50 = 100 (1 − e

RC )

−4
t = 0.69 × 10 sec.

Q4

  #MathBoleTohMathonGo
www.mathongo.com
Capacitance
  JEE Main 2021 (July) Chapter-wise Questions
Hints and Solutions MathonGo

1 d 2d 3d
= + +
Cef f K∈0 A 3K∈0 A 5K∈0 A

15K∈0 A
Ceff  =
34d

Q5

Now,

2CV + CV
VC =
KC + 2C

3V
=
K + 2

#MathBoleTohMathonGo
www.mathongo.com
Center
  of Mass Momentum and Collision JEE Main 2021 (July) Chapter-wise Questions
Questions with Answer Keys MathonGo

Q1 (20 July 2021 Shift 1)

A body having specific charge 8μC/g is resting on a frictionless plane at a distance 10 cm from the wall (as

shown in the figure). It starts moving

towards the wall when a uniform electric field of 100 V/m is applied horizontally towards the wall. If the

collision of the body with the wall is perfectly elastic, then the time period of the motion will be ___ s.

Q2 (20 July 2021 Shift 1)

A rod of mass M and length L is lying on a horizontal frictionless surface. A particle of mass 'm' travelling
along the surface hits at one end of the rod with a velocity 'u' in a direction perpendicular to the rod. The

collision is completely elastic. After collision, particle comes to rest. The ratio of masses ( m

M
) is 1

x
. The value

of ' x ' will be ___

Q3 (22 July 2021 Shift 1)

The position of the centre of mass of a uniform

semi-circular wire of radius 'R' placed in x -y plane with its centre at the origin and the line joining its ends as
-axis is given by (0, .

xR
x )
π

Then, the value of |x| is _____

Q4 (25 July 2021 Shift 1)

A body of mass 2 kg moving with a speed of 4 m/s. makes an elastic collision with another body at rest and
continues to move in the original direction but with one fourth of its initial speed. The speed of the two body
centre of mass is

10
x
 m/s . Then the value of x is

  #MathBoleTohMathonGo
www.mathongo.com
Center
  of Mass Momentum and Collision JEE Main 2021 (July) Chapter-wise Questions
Questions with Answer Keys MathonGo

Q5 (27 July 2021 Shift 1)

Three objects A, B and C are kept in a straight line on a frictionless horizontal surface. The masses of A, B

and C are m, 2 m and 2 m respectively. A moves towards B with a speed of 9 m/s and makes an elastic
collision with it. Thereafter B makes a completely inelastic collision with C. All motions occur along same
straight line. The final speed of C is :

(1) 6 m/s
(2) 9 m/s
(3) 4 m/s
(4) 3 m/s

  #MathBoleTohMathonGo
www.mathongo.com
Center
  of Mass Momentum and Collision JEE Main 2021 (July) Chapter-wise Questions
Questions with Answer Keys MathonGo

Answer Key

Q1 (1) Q2 (4) Q3 (2) Q4 (25)

Q5 (4)

#MathBoleTohMathonGo
www.mathongo.com
Center
  of Mass Momentum and Collision JEE Main 2021 (July) Chapter-wise Questions
Hints and Solutions MathonGo

Q1

F = ma

qE = ma

qE
a =
m

Now d = 1

2
at
2

t = √
2 d

2 d
t =
√ qE
( )
m

t =
√ 8×10−6
2×0.1
=
1

( )×100
10−3

∴ Time period = 2t = 1sec

Ans. = 1.00

Q2

Just before collision

  #MathBoleTohMathonGo
www.mathongo.com
Center
  of Mass Momentum and Collision JEE Main 2021 (July) Chapter-wise Questions
Hints and Solutions MathonGo

Just after collision

From momentum conservation, P 0


i
= Pf

mu = Mv … (i)

From angular momentum conservation about O,

mu ⋅
L

2
=
ML

12
ω

⇒ ω =
6mu

ML
...(ii)

From e =  R.V.S 

 R.V.A 

ωL
V+

1 =
u
2

v +
ωL

2
= u

v +
3mu

M
= u

mu

M
+
3mu

M
= u

4mu

M
= u

m 1
=
M 4

X = 4

Q3

COM of semi-circular ring is at 2R

  #MathBoleTohMathonGo
www.mathongo.com
Center
  of Mass Momentum and Collision JEE Main 2021 (July) Chapter-wise Questions
Hints and Solutions MathonGo

Distance from centre ⇒ x = 2

Q4

pi = pf

2 × 4 = 2 × 1 + m2 × v 2

m2  V2 = 6

v2 −1
by coefficient of restitution 1 = 4
⇒ v2 = 5 m/s

by (i) m 2 × 5 = 6

m2 = 1.2 kg

m1 v1 +m2 v2
vem =
 m1 +m2

2×1+1.2×5

8 25
vcm = = =
2+1.2 3.2 10

x = 25

Q5

Collision between A and B

m × 9 = mv1 + 2 mv2 (from momentum conservation)

v2 −v1
e = 1 =
9

⇒ v2 = 6 m/sec. , v1 = −3 m/sec

collision between B and C

2 m × 6 = 4mv (from momentum conservation)

v = 3 m/s

 
#MathBoleTohMathonGo
www.mathongo.com
Communication
  System JEE Main 2021 (July) Chapter-wise Questions
Questions with Answer Keys MathonGo

Q1 (20 July 2021 Shift 1)

A carrier wave V c (t) = 160sin (2π × 10 t)


6
volts is made to vary between V max = 200 V and V min = 120 V

by a message signal V m (t) = Am sin(2π × 10 t)


3
volts. The peak voltage A of the modulating signal is ___
m

Q2 (22 July 2021 Shift 1)

What should be the height of transmitting antenna and the population covered if the television telecast is to

cover a radius of 150 km ? The average population density around the tower is 2000/km and the value of 2

Re = 6.5 × 10  m
6
.

(1) Height = 1731 m

Population Covered = 1413 × 10 5

(2) Height = 1241 m

Population Covered = 7 × 10 5

(3) Height = 1600 m

Population Covered = 2 × 10 5

(4) Height = 1800 m

Population Covered = 1413 × 10 8

Q3 (25 July 2021 Shift 1)

In amplitude modulation, the message signal V m (t)


5
= 10 sin(2π × 10 t) volts and

Carrier signal V c (t) = 20 sin(2π × 10 t)


7
volts

The modulated signal now contains the message signal with lower side band and upper side band frequency,
therefore the bandwidth of modulated signal is αkHz. The value of α is :

(1) 200kHz
(2) 50kHz
(3) 100kHz
(4) 0

  #MathBoleTohMathonGo
www.mathongo.com
Communication
  System JEE Main 2021 (July) Chapter-wise Questions
Questions with Answer Keys MathonGo

Q4 (25 July 2021 Shift 2)

A message signal of frequency 20kHz and peak voltage of 20 volt is used to modulate a carrier wave of

frequency 1MHz and peak voltage of 20 volt. The modulation index will be

Q5 (27 July 2021 Shift 1)

The amplitude of upper and lower side bands of A.M. wave where a carrier signal with frequency 11.21MHz,
peak voltage 15 V is amplitude modulated by a 7.7kHz sine wave of 5 V amplitude are and
a b
 V  V
10 10

respectively. Then the value of is ___


a

Q6 (27 July 2021 Shift 2)

The maximum amplitude for an amplitude modulated wave is found to be 12 V while the minimum amplitude
is found to be 3 V. The modulation index is 0.6x where x is

  #MathBoleTohMathonGo
www.mathongo.com
Communication
  System JEE Main 2021 (July) Chapter-wise Questions
Questions with Answer Keys MathonGo

Answer Key

Q1 (40) Q2 (1) Q3 (1) Q4 (1)

Q5 (1) Q6 (1)

#MathBoleTohMathonGo
www.mathongo.com
Communication
  System JEE Main 2021 (July) Chapter-wise Questions
Hints and Solutions MathonGo

Q1

Maximum amplitude

Amax = Am + AC

⇒ Vmax = Vm + VC

200 = Vm + 160

Vm = 40

∴ Peak voltage A m = 40

Ans. 40

Q2

Radius covered r = √2RH


T

150 km = √2 × (6.5 × 10  m) HT


6

2
(150 km × 10 )
3
= 2 × 6.5 × 10 HT
6

HT = 1731 m

Population covered = (πr 2


) (2000/km )
2

2 5
= 3.14 × (150) × 2000 = 1413 × 10

Q3

Bandwidth = 2 × f
m

5
= 2 × 10 HZ = 200KHZ

Q4

Modulation index

Am 20
μ = = = 1
Ac 20

Q5

  #MathBoleTohMathonGo
www.mathongo.com
Communication
  System JEE Main 2021 (July) Chapter-wise Questions
Hints and Solutions MathonGo

μAC

a b
= =
10 10 2

a
⇒ = 1
b

Q6

Amax = Ac + Am = 12

Amin = Ac − Am = 3

⇒ Ac =
15

2
&Am =
9

Am 9/2
modulation index = Ac
=
15/2
= 0.6

⇒ x = 1

#MathBoleTohMathonGo
www.mathongo.com
Current
  Electricity JEE Main 2021 (July) Chapter-wise Questions
Questions with Answer Keys MathonGo

Q1 (20 July 2021 Shift 1)

The value of current in the 6Ω resistance is :

(1) 4 A

(2) 8 A
(3) 10 A
(4) 6 A

Q2 (20 July 2021 Shift 1)

A current of 5 A is passing through a non-linear magnesium wire of cross-section 0.04 m . At every point the
2

direction of current density is at an angle of 60 with the unit vector of area of cross-section. The magnitude of

electric field at every point of the conductor is :

(Resistivity of magnesium ρ = 44 × 10 −8
Ωm )

(1) 11 × 10 −2
 V/m

(2) 11 × 10 −7
 V/m

(3) 11 × 10 −5
 V/m

(4) 11 × 10 −3
 V/m

Q3 (20 July 2021 Shift 2)

In the given figure switches S and S are in open

1 2

condition. The resistance across ab when the

  #MathBoleTohMathonGo
www.mathongo.com
Current
  Electricity JEE Main 2021 (July) Chapter-wise Questions
Questions with Answer Keys MathonGo

switches S and S are closed is _____Ω

1 2

Q4 (22 July 2021 Shift 1)

A Copper (Cu) rod of length 25 cm and crosssectional area 3 mm is joined with a similar Aluminium (Al)
2

rod as shown in figure. Find the resistance of the combination between the ends A

and B. (Take Resistivity of Copper = 1.7 × 10 −8


Ωm Resistivity of Aluminium = 2.6 × 10 −8
Ωm )

(1) 2.170 mΩ
(2) 1.420 mΩ
(3) 0.0858 mΩ

(4) 0.858 mΩ

Q5 (22 July 2021 Shift 1)

In an electric circuit, a call of certain emf provides a potential difference of 1.25 V across a load resistance of
5Ω . However, it provides a potential

difference of 1 V across a load resistance of 2Ω

The emf of the cell is given by x

10
 V . Then the value of x is ____.

Q6 (25 July 2021 Shift 1)

  #MathBoleTohMathonGo
www.mathongo.com
Current
  Electricity JEE Main 2021 (July) Chapter-wise Questions
Questions with Answer Keys MathonGo

In the given figure, there is a circuit of potentiometer of length AB = 10 m. The resistance per unit length is
0.1Ω per cm. Across AB, battery of emf E and internal resistance 'r' is connected. The maximum value of emf
measured by this potentiometer is :

(1) 5 V
(2) 2.25 V

(3) 6 V
(4) 2.75 V

Q7 (25 July 2021 Shift 1)

An electric bulb rated as 200 W at 100 V is used in a circuit having 200 V supply. The resistance ' R that

must be put in series with the bulb so that the bulb delivers the same power is Ω.

Q8 (25 July 2021 Shift 2)

In the given potentiometer circuit arrangement, the balancing length AC is measured to be 250 cm. When the
galvanometer connection is shifted from point (1) to point (2) in the given diagram, the balancing length

becomes 400 cm. The ratio of the

ε1
emf of two cells, ε.
is

(1)
5

  #MathBoleTohMathonGo
www.mathongo.com
Current
  Electricity JEE Main 2021 (July) Chapter-wise Questions
Questions with Answer Keys MathonGo

(2)
8

(3) 4

(4)
3

Q9 (25 July 2021 Shift 2)

The given potentiometer has its wire of resistance

10Ω . When the sliding contact is in the middle of

the potentiometer wire, the potential drop across

2Ω resistor is :

(1) 10 V

(2) 5 V
(3) 40

9
 V

(4)
40
 V
11

Q10 (25 July 2021 Shift 2)

A 16Ω wire is bend to form a square loop. A 9 V supply having internal resistance of 1Ω is connected across
one of its sides. The potential drop across the diagonals of the square loop is ×10 −1
 V

Q11 (27 July 2021 Shift 1)

In the given figure, a battery of emf E is connected across a conductor PQ of length ' l and different area of

cross-sections having radii r and r


1 2 (r2 < r1 ) .

  #MathBoleTohMathonGo
www.mathongo.com
Current
  Electricity JEE Main 2021 (July) Chapter-wise Questions
Questions with Answer Keys MathonGo

Choose the correct option as one moves from P to Q :

(1) Drift velocity of electron increases.


(2) Electric field decreases.

(3) Electron current decreases.


(4) All of these

Q12 (27 July 2021 Shift 2)

The resistance of a conductor at 15 ∘


C is 16Ω and

at 100 ∘
C is 20Ω. What will be the temperature

coefficient of resistance of the conductor?

(1) 0.010 ∘
C
−1

(2) 0.033 ∘
C
−1

(3) 0.003 ∘
C
−1

(4) 0.042 ∘
C
−1

Q13 (27 July 2021 Shift 2)

For the circuit shown, the value of current at time

t = 3.2 s will be A .

  #MathBoleTohMathonGo
www.mathongo.com
Current
  Electricity JEE Main 2021 (July) Chapter-wise Questions
Questions with Answer Keys MathonGo

[Voltage distribution V(t) is shown by Fig. (1) and

the circuit is shown in Fig. (2)]

  #MathBoleTohMathonGo
www.mathongo.com
Current
  Electricity JEE Main 2021 (July) Chapter-wise Questions
Questions with Answer Keys MathonGo

Answer Key

Q1 (3) Q2 (3) Q3 (10) Q4 (4)

Q5 (15) Q6 (1) Q7 (50) Q8 (1)

Q9 (3) Q10 (45) Q11 (1) Q12 (3)

Q13 (1)

#MathBoleTohMathonGo
www.mathongo.com
Current
  Electricity JEE Main 2021 (July) Chapter-wise Questions
Hints and Solutions MathonGo

Q1

V−0 V−90 V−140


Applying KCL at point P, 6
+
5
+
20
= 0

⇒ 10 V + 12 V − 1080 + 3 V − 420 = 0

⇒ V = 60

V−0
∴ current in 6Ω = 6
= 10 A

Hence option 3 .

Q2
→ →
I = J ⋅ A = JA cos(θ)

5 = J(
4

100
) × cos(60)

2
J = 5 × 50 = 250 A/m

→ →
Now, E = ρ ⋅ J

−8 −5
= 44 × 10 × 250 = 11 × 10  V/m

Q3

When switch S and S are closed

1 2

12×6 6×12
+ 2 +
12+6 6+12

72 72
+ 2 + = 4 + 2 + 4 = 10Ω
18 18

  #MathBoleTohMathonGo
www.mathongo.com
Current
  Electricity JEE Main 2021 (July) Chapter-wise Questions
Hints and Solutions MathonGo

Q4
R1 R2 ρ1 ρ2
R =
R1 +R2
=
A


ρ1 +ρ2

−2 −16

25×10 1.7×2.6×10
R = −6
× −8
3×10 4.3×10

R = 0.858 mΩ

Q5

Terminal voltage v = iR =

ER

R+r

E(5)
1
st
→ 1.25 =
5+r
. . (i)

E(2)
2
nd
→ 1 =
2+r
… (ii)

By (i) and (ii)

r = 1ΩE =
3

2
 V =
15

10
volt

⇒ x = 15

Q6

Max, voltage that can be measured by this potentiometer will be equal to potential drop across AB
RAB = 10 × 0.1 × 100 = 100ohm.

6 100
VAB = × 100 = 6 × = 5 V
20+100 120

Q7

  #MathBoleTohMathonGo
www.mathongo.com
Current
  Electricity JEE Main 2021 (July) Chapter-wise Questions
Hints and Solutions MathonGo
2

Power P = V

RB

RB =
V

P
=
100×100

200

RB = 50Ω

To produce same power, same voltage (i.e. 100 V ) should be across the bulb Hence, R = R
B

R = 50Ω

Q8

E1 = kℓ1 ...(i)

E1 + E2 = kℓ2 … (ii)

E1 ℓ1

E1 +E2
=
ℓ2
=
250

400
=
5

8E1 = 5E1 + 5E2

3E1 = 5E2

E1 5
=
E2 3

Q9

20−V0 0−V0 20−V0

5
+
5
+
2
= 0

2 V0 V0
4 + 10 =
5
+
2

4 V0 +5 V0
14 =
10

140
V0 = Volt
9

  #MathBoleTohMathonGo
www.mathongo.com
Current
  Electricity JEE Main 2021 (July) Chapter-wise Questions
Hints and Solutions MathonGo

Potential difference across 2Ω resistor is 20 − V That is (20 − Volt

140
0 )
9

Hence answer is ( Volt


40
)
9

Q10

here assume current as

By KVL in outer loop 9 − 12i − 4i = 0

9
8i = = 4.5
2

−1
= 45 × 10

Q11

Current is constant in conductor i = constant

ρdx
Resistance of element dR = πr
2

ipdx
dV = idR = 2

πr


dV
E = = 2
dx πr

eEτ
& Vd =
m

∴ Vd ∝ E

1
→ E ∝ 2
r

if r decreases, E will increase ∴ V will increase d

  #MathBoleTohMathonGo
www.mathongo.com
Current
  Electricity JEE Main 2021 (July) Chapter-wise Questions
Hints and Solutions MathonGo

Q12

16 = Ro [1 + α (15 − To )]

20 = Ro [1 + α (100 − To )]

Assuming T o = 0 C

, as a general convention.

1+α×15

16
⇒ =
20 1+α×100

∘ −1
⇒ α = 0.003 C

Q13

From graph voltage at t = 3.2sec is 6 volt.

6−5
i =
1

i = 1 A

#MathBoleTohMathonGo
www.mathongo.com
Dual
  Nature of Matter JEE Main 2021 (July) Chapter-wise Questions
Questions with Answer Keys MathonGo

Q1 (20 July 2021 Shift 2)

An electron having de-Broglie wavelength λ is

incident on a target in a X-ray tube. Cut-off wavelength of emitted X-ray is:

(1) 0
2 2 2

(2) 2 m c λ

2
 h
2

(3) 2mcλ

 h

(4) hc

mc

Q2 (20 July 2021 Shift 2)

A certain metallic surface is illuminated by

monochromatic radiation of wavelength λ. The

stopping potential for photoelectric current for this radiation is 3 V . If the same surface is illuminated with a
0

radiation of wavelength 2λ, the stopping potential is V . The threshold wavelength of this

surface for photoelectric effect is ____ λ

Q3 (22 July 2021 Shift 1)

An electron of mass m and a proton of mass m are accelerated through the same potential difference. The
e P

ratio of the de-Broglie wavelength associated with the electron to that with the proton is :-
mP
(1) me

(2) 1
mp
(3) √ me

me
(4)  mp

Q4 (25 July 2021 Shift 1)

What should be the order of arrangement of de-Broglie wavelength of electron (λ ), an α -particle (λ


e α) and
proton (λ p) given that all have the same kinetic energy ?

(1) λ e = λp = λα

  #MathBoleTohMathonGo
www.mathongo.com
Dual
  Nature of Matter JEE Main 2021 (July) Chapter-wise Questions
Questions with Answer Keys MathonGo

(2) λ e < λp < λα

(3) λ e > λp > λα

(4) λ e = λp > λα

Q5 (25 July 2021 Shift 1)

A particle of mass 4M at rest disintegrates into two particles of mass M and 3M respectively having non zero

velocities. The ratio of de-Broglie wavelength of particle of mass M to that of mass 3M will be :

(1) 1: 3
(2) 3: 1

(3) 1 : √3
(4) 1: 1

Q6 (25 July 2021 Shift 2)

An electron moving with speed v and a photon moving with speed c, have same D -Broglie wavelength. The
ratio of kinetic energy of electron to that of photon is:

(1) 3c

(2) v

3c

(3) v

2c

(4)
2c

Q7 (25 July 2021 Shift 2)

When radiation of wavelength λ is incident on a metallic surface, the stopping potential of ejected
photoelectrons is 4.8 V. If the same surface is illuminated by radiation of double the previous wavelength,
then the stopping potential becomes 1.6 V. The threshold wavelength of the metal is :

(1) 2λ
(2) 4λ
(3) 8λ

  #MathBoleTohMathonGo
www.mathongo.com
Dual
  Nature of Matter JEE Main 2021 (July) Chapter-wise Questions
Questions with Answer Keys MathonGo

(4) 6λ

Q8 (25 July 2021 Shift 2)

A light beam of wavelength 500 nm is incident on a metal having work function of 1.25eV, placed in a
magnetic field of intensity B. The electrons emitted perpendicular to the magnetic field B, with maximum
kinetic energy are bent into circular arc of radius 30 cm. The value of B is ×10 −7
 T

Given hc = 20 × 10 −26
 J − m , mass of electron

−31
= 9 × 10  kg

Q9 (27 July 2021 Shift 1)

A particle of mass 9.1 × 10 −31


 kg travels in a medium with a speed of 10 6
 m/s and a photon of a radiation of
linear momentum 10 −27
 kg m/s travels in vacuum. The wavelength of photon is ___ times the wavelength of
the particle.

Q10 (27 July 2021 Shift 2)

An electron and proton are separated by a large distance. The electron starts approaching the proton with
energy 3eV. The proton captures the electrons and forms a hydrogen atom in second excited state. The

resulting photon is incident on a photosensitive metal of threshold wavelength 4000Ã …. What is the
maximum kinetic energy of the emitted photoelectron?

(1) 7.61eV

(2) 1.41eV
(3) 3.3eV

(4) No photoelectron would be emitted

  #MathBoleTohMathonGo
www.mathongo.com
Dual
  Nature of Matter JEE Main 2021 (July) Chapter-wise Questions
Questions with Answer Keys MathonGo

Answer Key

Q1 (3) Q2 (4) Q3 (3) Q4 (3)

Q5 (4) Q6 (3) Q7 (2) Q8 (125)

Q9 (910) Q10 (2)

#MathBoleTohMathonGo
www.mathongo.com
Dual
  Nature of Matter JEE Main 2021 (July) Chapter-wise Questions
Hints and Solutions MathonGo

Q1

h
λ =
mv

2 2

kinetic energy,

P h hc
= 2
=
2 m 2 mλ λc

2
2 mλ c
λC =
h

Q2

hc
KE = − ϕhc
λ

e (3 V0 ) =
hc

λ0
− ϕ

eV0 =
hc

2λ0
− ϕ

Using (i) & (ii)

ϕ =
hc

4λ0
=
hc

λt

λt = 4λ0

Q3

KE = eΔV

λe =
h

√2 me (eΔV)

λP =
h

√2 mp (eΔV)

λe mP
⇒ = √
λP me

Q4

Sol.

h h 1
λ = = ∝
p
√2mE √m

mα > mp > me

so λ e > λp > λα

Q5

λ =
h

both the particles will move with momentum same in magnitude \& opposite in direction. So De-Broglie

wavelength of both will be same i.e. ratio 1 : 1

  #MathBoleTohMathonGo
www.mathongo.com
Dual
  Nature of Matter JEE Main 2021 (July) Chapter-wise Questions
Hints and Solutions MathonGo

Q6

λe = λFh

h h
=
p p
e ph

Eph
√2mke =
c

2
(Eph )
2mke =
c
2

ke Eph

Eph
=
c
2
(
2 m
1
)

pph

1
= ( )
c 2 m

e 1
= ( )
c 2 m

=
mv

c 2 m
1

v
=
2c

Q7

VS = hv − ϕ ...(i)

4.8 =
hc

λ
− ϕ ...(ii)

1.6 =
hc


− ϕ … (iii)

Using above equation (i) - (ii) 3.2 = hc

λ

hc


3.2 =
hc


… (iii)

[λ =
6.4
hc
]

Put in equation (ii) ϕ = 1.6

λth
hc
= 1.6

λth  =
 hc 

1.6

hc
= ( ) × 4 = 4λ
6.4

Q8

By photoelectric equation

hc

λ
− ϕ = kmax

kmax =
1240

500
− 1.25 ≈ 1.25

√2mk
r =
eB

  #MathBoleTohMathonGo
www.mathongo.com
Dual
  Nature of Matter JEE Main 2021 (July) Chapter-wise Questions
Hints and Solutions MathonGo
√2mk
B =
er

−7
= 125 × 10  T

Q9
−34

For photon λ1 =
h

P
=
6.6×10

−27

10
−34

For particle λ 2 =
h

mv
=
6.6×10

−31 6

9.1×10 ×10

λ1
∴ = 910
λ2

Q10
(13.6eV)
Initially, energy of electron = +3eV finally, in 2 nd 
excited state, energy of electron = − 2

= −1.51eV

Loss in energy is emitted as photon, So, photon energy hc

λ
= 4.51eV

hc hc
KEmax = − ϕ = 4.51 − ( )
λ λth 

Now, photoelectric effect equation 12400eV Ã…


= 4.51eV −
4000 Ã…
= 1.41eV

#MathBoleTohMathonGo
www.mathongo.com
Electromagnetic
  Induction JEE Main 2021 (July) Chapter-wise Questions
Questions with Answer Keys MathonGo

Q1 (20 July 2021 Shift 1)

The arm PQ of a rectangular conductor is moving from x = 0 to x = 2b outwards and then inwards from

x = 2b to x = 0 as shown in the figure. A uniform magnetic field perpendicular to the plane is acting from

x = 0 to x = b. Identify the graph showing the variation of different quantities with distance :

(1) A-Flux, B-Power dissipated, C-EMF

(2) A-Power dissipated, B-Flux, C-EMF


(3) A-Flux, B-EMF, C-Power dissipated
(4) A-EMF, B-Power dissipated, C-Flux

Q2 (25 July 2021 Shift 1)



A circular conducting coil of radius 1 m is being heated by the change of magnetic field B passing
perpendicular to the plane in which the coil is laid. The resistance of the coil is 2μΩ. The magnetic field is

slowly switched off such that its magnitude changes in time as B =

4 −3 t
× 10  T (1 − )
π 100

The energy dissipated by the coil before the magnetic field is switched off completely is E = mJ.

Q3 (27 July 2021 Shift 1)

  #MathBoleTohMathonGo
www.mathongo.com
Electromagnetic
  Induction JEE Main 2021 (July) Chapter-wise Questions
Questions with Answer Keys MathonGo

Consider an electrical circuit containing a two way switch 'S'. Initially S is open and then T is connected to
1

T2 . As the current in R = 6Ω attains a maximum value of steady state level, T is disconnected from T and
1 2

immediately connected to T . Potential drop across r = 3Ω resistor immediately after T is connected to T is


3 1 3

___ V.

(Round off to the Nearest Integer)

Q4 (27 July 2021 Shift 2)

In the given figure the magnetic flux through the loop increases according to the relation ϕ B (t) = 10t
2
+ 20t ,
where ϕ is in milliwebers and t

is in seconds. The magnitude of current through R = 2Ω resistor at t = 5 s is

mA

  #MathBoleTohMathonGo
www.mathongo.com
Electromagnetic
  Induction JEE Main 2021 (July) Chapter-wise Questions
Questions with Answer Keys MathonGo

Answer Key

Q1 (3) Q2 (80) Q3 (3) Q4 (60)

#MathBoleTohMathonGo
www.mathongo.com
Electromagnetic
  Induction JEE Main 2021 (July) Chapter-wise Questions
Hints and Solutions MathonGo

Q1

As rod moves in field area increases upto x̄ = b nthen field is absent and again flux is generated on return

journey from x = b to x = 0. Thus plot A for flux.


⇒ e = −
dt
⇒ curve B for emf

⇒ Power dissipated = vi ⇒ curve C for power dissipated

Q2

ϕ = BS → →

ϕ =
4

π
× 10
−3
(1 −
t

100
) ⋅ πR
2

−3 2 t
ϕ = 4 × 10 × (1) (1 − )
100

−dϕ
ε =
dt

ε =
−d

dt
(4 × 10
−3
(1 −
t

100
))

ε = 4 × 10
−3
(
100
1
) = 4 × 10
−5
 V

When B = 0 1 − t

100
= 0

t = 100sec

Heat = ε

R
t

2
−5
(4×10 )

Heat = −6
× 100 J

2×10
−10

Heat = 16×10 ×100

−6
 J

2×10

Heat = 0.08 J

Heat = 80 mJ

Q3

When T and T are connected, then the steady state current in the inductor I

6
1 2
= = 1 A
6

When T and T are connected then current through inductor remains same. So potential difference across 3Ω

1 3

V = Ir = 1 × 3 = 3 volt 

Q4

  #MathBoleTohMathonGo
www.mathongo.com
Electromagnetic
  Induction JEE Main 2021 (July) Chapter-wise Questions
Hints and Solutions MathonGo


|ϵ| =
dt
= 20t + 20mV

|∈|
|i| =
R
= 10t + 10 mA

at t = 5

|i| = 60 mA

#MathBoleTohMathonGo
www.mathongo.com
Electromagnetic
  Waves JEE Main 2021 (July) Chapter-wise Questions
Questions with Answer Keys MathonGo

Q1 (20 July 2021 Shift 2)

In an electromagnetic wave the electric field vector


and magnetic field vector are given as E = E0 ^
i


and ^
B = B0 k respectively. The direction of propagation of electromagnetic wave is along :

(1) (k)
^

(2) J^

(3) (−k)
^

(4) (−^j)

Q2 (22 July 2021 Shift 1)

Intensity of sunlight is observed as 0.092Wm −2


at a point in free space. What will be the peak value

of magnetic field at that point? (ε 0


= 8.85 × 10
−12 2
C  N
−1
 m
−2
)

(1) 2.77 × 10 −8
 T

(2) 1.96 × 10 −8
 T

(3) 8.31 T
(4) 5.88 T

Q3 (25 July 2021 Shift 1)

A linearly polarized electromagnetic wave in

vacuum is

6
E = 3.1 cos[(1.8)z − (5.4 × 10 ) t]iN/C

is incident normally on a perfectly reflecting wall at z = a. Choose the correct option

(1) The wavelength is 5.4 m


(2) The frequency of electromagnetic wave is 54 × 10 4
 Hz

(3) The transmitted wave will be

6
3.1 cos[(1.8)z − (5.4 × 10 ) t]^
i N/C

(4) The reflected wave will be 1 cos[(1.8)z + (5.4 × 10 6


) t]^
i N/C

  #MathBoleTohMathonGo
www.mathongo.com
Electromagnetic
  Waves JEE Main 2021 (July) Chapter-wise Questions
Questions with Answer Keys MathonGo

Answer Key

Q1 (4) Q2 (1) Q3 (4)

#MathBoleTohMathonGo
www.mathongo.com
Electromagnetic
  Waves JEE Main 2021 (July) Chapter-wise Questions
Hints and Solutions MathonGo

Q1
→ →
Direction of propagation = E × B = ^ ^ = −^
i × k j

Q2
2
B C

0 1 2
Iavg = & = ϵ0 C
2μ0 μ0
2
B
I =
2
0
∈0 C
3

 B0 = √
2I

ϵ0 C

−8
 B0 = 2.77 × 10  T

Q3

Reflected wave will have direction opposite to

incident wave.

#MathBoleTohMathonGo
www.mathongo.com
Electrostatics
  JEE Main 2021 (July) Chapter-wise Questions
Questions with Answer Keys MathonGo

Q1 (20 July 2021 Shift 1)

A certain charge Q is divided into two parts q and (Q-q). How should the charges Q and q be divided so that q

and (Q-q) placed at a certain distance apart experience maximum electrostatic repulsion ?
q
(1) Q = 2

(2) Q = 2q

(3) Q = 4q

(4) Q = 3q

Q2 (22 July 2021 Shift 1)

An electric dipole is placed on x -axis in proximity to a line charge of linear charge density 3.0 × 10 −6
C/m .

Line charge is placed on z -axis and positive and negative charge of dipole is at a distance of 10 mm and
12 mm from the origin respectively. If total force of 4 N is exerted on the dipole, find out the amount of

positive or negative

charge of the dipole.

(1) 815.1nC
(2) 8.8μC

(3) 0.485mC
(4) 4.44μC

Q3 (22 July 2021 Shift 1)

The total charge enclosed in an incremental

volume of 2 × 10 −9
 m
3
located at the origin is ____ nC, if electric flux density of its field is found as

D = e
−x
siny ^
i − e
−x
cosy ^ ^
j + 2zkC/m
2
.

Q4 (25 July 2021 Shift 1)

A particle of mass 1mg and charge q is ly ing at the mid-point of two stationary particles kept at a distance '
2 m ' when each is carrying same charge ′
q .

If the free charged particle is displaced from its equilibrium

  #MathBoleTohMathonGo
www.mathongo.com
Electrostatics
  JEE Main 2021 (July) Chapter-wise Questions
Questions with Answer Keys MathonGo

position through distance ' x ′


(x << 1 m) . The particle executes SHM. Its angular frequency of oscillation
will be ×10 5
rad/s

if q 2
= 10C
2

Q5 (25 July 2021 Shift 2)

Two ideal electric dipoles A and B, having their dipole moment p and p respectively are placed on a plane
1 2

with their centres at O as shown in the figure. At point C on the axis of dipole A, the resultant electric field is
p
making an angle of 37 with the axis. The ratio of the dipole moment of A and B, is : ( take sin 37 )

∘ 1 ∘ 3
=
p 5
2

(1) 3

(2) 3

(3) 2

(4)
4

Q6 (27 July 2021 Shift 1)

The relative permittivity of distilled water is 81 . The velocity of light in it will be:

( Given μr = 1)

(1) 4.33 × 10 7
 m/s

(2) 2.33 × 10 7
 m/s

(3) 3.33 × 10 7
 m/s

(4) 5.33 × 10 7
 m/s

Q7 (27 July 2021 Shift 2)

A simple pendulum of mass ' m ', length ' l and charge '+q' suspended in the electric field produced by two
conducting parallel plates as shown. The value of deflection of pendulum in equilibrium position will be

  #MathBoleTohMathonGo
www.mathongo.com
Electrostatics
  JEE Main 2021 (July) Chapter-wise Questions
Questions with Answer Keys MathonGo

q C1 ( V2 −V1 )
(1) tan −1
[
mg
×
(C1 +C2 )(d−t)
]

q C2 ( V2 −V1 )
(2) tan −1
[
mg
×
(C1 +C2 )(d−t)
]

q C2 ( V1 +V2 )
(3) tan −1
[
mg
×
(C1 +C2 )(d−t)
]

q C1 ( V1 +V2 )
(4) tan −1
[
mg
×
(C1 +C2 )(d−t)
]

Q8 (27 July 2021 Shift 2)

What will be the magnitude of electric field at point O as shown in figure? Each side of the figure is l and
perpendicular to each other?

q
(1)
1

4πε0 l
2

q
(2) 1

4πε0 (2l )
2
(2√2 − 1)

q
(3) 4πε0 (2l)
2

2q
(4)
1

2
(√2)
4πε0 2l

  #MathBoleTohMathonGo
www.mathongo.com
Electrostatics
  JEE Main 2021 (July) Chapter-wise Questions
Questions with Answer Keys MathonGo

Answer Key

Q1 (2) Q2 (4) Q3 (4) Q4 (6)

Q5 (3) Q6 (3) Q7 (3) Q8 (2)

#MathBoleTohMathonGo
www.mathongo.com
Electrostatics
  JEE Main 2021 (July) Chapter-wise Questions
Hints and Solutions MathonGo

Q1

kq(Q−q)
Fq = 2
=
k

2
(qQ − q )
2

L L

dF

dq
= 0 when force is maximum

dF

dq
=
k

2
[Q − 2q] = 0

⇒ Q − 2q = 0 ⇒ Q = 2q

Q2

r = 10 mm, x = 2,


∣ ∣ 2kλ
∣ F q∣ = r
⋅ q
∣ ∣


∣ ∣ 2kλ
∣ F −q ∣ = r+x
⋅ q
∣ ∣


∣ ∣ 2kλq 2kλq
⇒ ∣ F net ∣ = −
r r+x
∣ ∣


∣ ∣ 2kλq⋅x
∣ F net  ∣ =
∣ ∣ r(r+x)

9 −6
2×9×10 ×3×10 ×q×2 mm
4 =
10 mm.12 mm

⇒ q = 4.44μC

Q3

  #MathBoleTohMathonGo
www.mathongo.com
Electrostatics
  JEE Main 2021 (July) Chapter-wise Questions
Hints and Solutions MathonGo

Electric flux density

→  charge  Q Q
(D) =
 Area 
× ^
r =
4πr
2
r = ϵ0 (
^
4πϵ0 r
2
r)
^



−x ^ −x ^
e sin y i −e ^+2zk
cos y
⇒ E =
D

ϵ0
=
ϵ0

Also by Gauss's law


ρ

ϵ0
= (

∂x
^
i +

∂y
^
j +
∂z

^
k) ⋅ E = (

∂x
^
i +

∂y
^
j +
∂z

^
k) ⋅
D

ϵ0

∂ −x ∂ −x ∂
⇒ ρ = (e sin y) + (−e cos y) + (2z)
∂x ∂y ∂z

ρ = −e
−x
sin y + e
−x
sin y + 2

At origin ρ = −e −0
sin 0 + e
−0
sin 0 + 2

ρ = 2C/m
3

Charge = ρ× volume = 2 × 2 × 10 −9
= 4 × 10
−9
=

4nC

Q4

2 2
kq kq
Net force on free charged particle F = ( d+x)
2

( d−x)
2

F = −kq
2
[
4 dx

2
]

2 2
(d −x )

2
4kq d
a = −
m
(
d
x
4
)

2
4kq
a = −( 3
)x

md

2
4kq
So, angular frequency ω = √ 3

md

ω = √
4×9×10 ×10
−6 3

1×10 ×1

8
ω = 6 × 10 rad/sec

Q5

  #MathBoleTohMathonGo
www.mathongo.com
Electrostatics
  JEE Main 2021 (July) Chapter-wise Questions
Hints and Solutions MathonGo

kP2

P2
tan 37

=
3

4
=
r3

2kP
1
=
2P1
=
3

r3

P2

P1
=
3

P1 2
=
P2 3

Q6

c
V =
√μr εr

7
= 3.33 × 10  m/sec

Q7

Let E be electric field in air T sin θ = qE

T cos θ = mg

qE
tan θ =
mg

C1 C2
Q = [
C1 +C2
] [V1 + V2 ]

  #MathBoleTohMathonGo
www.mathongo.com
Electrostatics
  JEE Main 2021 (July) Chapter-wise Questions
Hints and Solutions MathonGo
Q C1 C2 [V1 +V2 ]
E =
Aϵo
= [
C1 +C2
]
Aϵo

ϵo A C2 [ V1 +V2 ]
C1 =
d−t
⇒ E =
(C1 +C2 )(d−t)

q⋅E
Now θ = tan −1
[
mg
]

q C2 ( V1 +V2 )
−1
θ = tan [ × ]
mg (C1 +C2 )(d−t)

Q8
kq
E1 =

2
= E2

kq kq
E3 =
2
=
2ℓ
2

(√2ℓ)

√2kq kq kq
E =

2

2ℓ
2
=
2ℓ
2
(2√2 − 1)

#MathBoleTohMathonGo
www.mathongo.com
Experimental
  Physics JEE Main 2021 (July) Chapter-wise Questions
Questions with Answer Keys MathonGo

Q1 (25 July 2021 Shift 1)

Student A and Student B used two screw gauges of

equal pitch and 100 equal circular divisions to

measure the radius of a given wire. The actual

value of the radius of the wire is 0.322 cm. The

absolute value of the difference between the final

circular scale readings observed by the students A

and B is

[Figure shows position of reference 'O' when jaws

of screw gauge are closed]

Given pitch = 0.1 cm.

Q2 (27 July 2021 Shift 1)

Assertion A : If in five complete rotations of the circular scale, the distance travelled on main scale of the

screw gauge is 5 mm and there are 50 total divisions on circular scale, then least count is 0.001 cm.

Reason R :

Least Count =  Pitch 

 Total divisions on circular scale 


In the light of the above statements, choose the most appropriate answer from the options given below :

(1) A is not correct but R is correct.


(2) Both A and R are correct and R is the correct

explanation of A.
(3) A is correct but R is not correct.

  #MathBoleTohMathonGo
www.mathongo.com
Experimental
  Physics JEE Main 2021 (July) Chapter-wise Questions
Questions with Answer Keys MathonGo

(4) Both A and R are correct and R is NOT the

correct explanation of A.

  #MathBoleTohMathonGo
www.mathongo.com
Experimental
  Physics JEE Main 2021 (July) Chapter-wise Questions
Questions with Answer Keys MathonGo

Answer Key

Q1 (13) Q2 (1)

#MathBoleTohMathonGo
www.mathongo.com
Experimental
  Physics JEE Main 2021 (July) Chapter-wise Questions
Hints and Solutions MathonGo

Q1

For (A)

Read ing = MSR + CSR+ Error

0.322 = 0.300 + CSR + 5 × LC

0.322 = 0.300 + CSR + 0.005

CSR = 0.017

For B

Reading = MSR + CSR+ Error 0.322 = 0.200 + CSR + 0.092

CSR = 0.030

Difference = 0.030 − 0.017 = 0.013 cm

Division on circular scale = 0.013

0.001
= 13

Q2

Least count =  Pitch 

 total division on circular scale 


In 5 revolution, distance travel, 5 mm In 1 revolution, it will travel 1 mm.

So least count = 1

50
= 0.02

#MathBoleTohMathonGo
www.mathongo.com
Gravitation
  JEE Main 2021 (July) Chapter-wise Questions
Questions with Answer Keys MathonGo

Q1 (20 July 2021 Shift 1)

A person whose mass is 100 kg travels from Earth to Mars in a spaceship. Neglect all other objects in sky and
take acceleration due to gravity on the surface of the Earth and Mars as 10 m/s and 4 m/s respectively.
2 2

Identify from the below figures, the curve that fits best for the weight of the passenger as a function of time.

(1) (c)
(2) (a)

(3) (d)
(4) (b)

Q2 (20 July 2021 Shift 2)

A satellite is launched into a circular orbit of radius R around earth, while a second satellite is launched into a

circular orbit of radius 1.02R. The percentage difference in the time periods of the

two satellites is :

(1) 1.5
(2) 2.0
(3) 0.7
(4) 3.0

Q3 (20 July 2021 Shift 2)

Consider a binary star system of star A and star B with masses m and m revolving in a circular orbit of radii
A B

rA and r , respectively. If T and T are the time period of star A and star B, respectively, then :
B A B

TA rA
(1)
2

= ( )
TB rB

  #MathBoleTohMathonGo
www.mathongo.com
Gravitation
  JEE Main 2021 (July) Chapter-wise Questions
Questions with Answer Keys MathonGo

(2) T A
= TB

(3) T A
> TB ( if mA > mB )

(4) T A
> TB (if rA > rB )

Q4 (22 July 2021 Shift 1)

A body is projected vertically upwards from the

surface of earth with a velocity sufficient enough

to carry it to infinity. The time taken by it to reach

height h is

S .
3/2
Re
(1) √ 2 g
[(1 +
h

Re
) − 1]

3/2
2Re
(2) √ g
[(1 +
h

Re
) − 1]

3/2
Re
(3)
1 h
√ [(1 + ) − 1]
3 2 g Re

3/2
2Re
(4) 1

3

g
[(1 +
h

Re
) − 1]

Q5 (25 July 2021 Shift 1)

The minimum and maximum distances of a planet revolving around the Sun are x and x . If the minimum 1 2

speed of the planet on its trajectory is v then its maximum speed will be:
0

2
v0 x

(1) x
2
1

2
2
V0 X
(2)
2

2
x
1

v0 x1
(3) x2

v0 x2
(4) x1

Q6 (25 July 2021 Shift 2)

Consider a planet in some solar system which has a mass double the mass of earth and density equal to the

average density of earth. If the weight of an object on earth is W, the weight of the same object on that planet

  #MathBoleTohMathonGo
www.mathongo.com
Gravitation
  JEE Main 2021 (July) Chapter-wise Questions
Questions with Answer Keys MathonGo

will be :

(1) 2 W

(2) W
1

(3) 2 3
 W

(4) √2 W

Q7 (27 July 2021 Shift 1)

Suppose two planets (spherical in shape) of radii R and 2R, but mass M and 9M respectively have a centre to
centre separation 8R as shown in the figure. A satellite of mass 'm' is projected from the surface of the planet
of mass 'M' directly towards the centre of the second planet. The minimum speed 'v' required for the satellite to
reach the surface of the second planet is √ a

7
GM

R
then the value of ' a ' is ___

[Given : The two planets are fixed in their position]

Q8 (27 July 2021 Shift 2)

Two identical particles of mass 1 kg each go round a circle of radius R, under the action of their mutual
gravitational attraction. The angular speed of each particle is :

(1) √
G

3
2R

(2)
1 G
√ 3
2 R

(3) 1

2R

1

(4) √
2G

3
R

Q9 (27 July 2021 Shift 2)

  #MathBoleTohMathonGo
www.mathongo.com
Gravitation
  JEE Main 2021 (July) Chapter-wise Questions
Questions with Answer Keys MathonGo

The planet Mars has two moons, if one of them has

a period 7 hours, 30 minutes and an orbital radius of 9.0 × 10 3


 km. Find the mass of Mars.

Given
4π 11 −1 −2 2
{ = 6 × 10  N  m  kg }
G

(1) 5.96 × 10 19
 kg

(2) 3.25 × 10 21
 kg

(3) 7.02 × 10 25
 kg

(4) 6.00 × 10 23
 kg

  #MathBoleTohMathonGo
www.mathongo.com
Gravitation
  JEE Main 2021 (July) Chapter-wise Questions
Questions with Answer Keys MathonGo

Answer Key

Q1 (1) Q2 (4) Q3 (2) Q4 (4)

Q5 (4) Q6 (3) Q7 (4) Q8 (2)

Q9 (4)

#MathBoleTohMathonGo
www.mathongo.com
Gravitation
  JEE Main 2021 (July) Chapter-wise Questions
Hints and Solutions MathonGo

Q1

At neutral point g = 0 so graph (C) is correct

Hence option (1).

Q2

T
2
∝ R
3

 T = kR
3/2

dT

T
=
3

2
dR

=
3

2
× 0.02 = 0.03

% Change = 3%

Q3

TA = TB ( since ωA = ωB )

Q4

  #MathBoleTohMathonGo
www.mathongo.com
Gravitation
  JEE Main 2021 (July) Chapter-wise Questions
Hints and Solutions MathonGo

 Applying energy conservation from (1) to (2) 

1 2GM GMm 1 2
GMm
 m ⋅ ( ) − = mv −
2 Re Re 2 R + r

1 2
GMm
⇒ mv =
2 R + r

2GM dr
⇒ v = √ =
R + r dt

t Re th

⇒√2GM ∫ dt = ∫ (√R + r)dr


0 Re

Re +h
2 3/2
√2GM ⋅ t = [(R + r) ]
3 Re

3 3/2
2 Re h
t = √ [(1 + ) − 1]
3 2GM Re

GM
= g
2
Re

3/2
1 2Re h
t = √ [(1 + ) − 1]
3  g Re

Q5

Angular momentum conservation equation v 0 x2 = v1 x1

v0 x2
v1 =
x1

Q6

Density is same M = 4

3
3
πR ρ, 2 m =
4

3
πR
33
ρ

R

= 2
1/3
R

GMm
ω = 2
R

ω2 =
G2Mm

′2

1/3
ω2 = 2 ω

Q7

  #MathBoleTohMathonGo
www.mathongo.com
Gravitation
  JEE Main 2021 (July) Chapter-wise Questions
Hints and Solutions MathonGo

Acceleration due to gravity will be zero at P therefore,

GM G9M

2
= 2
x (8R−x)

8R − x = 3x

x = 2R

Apply conservation of energy and consider velocity at P is zero.

2
mv
2

GMm

R

G9Mm

7R
= 0 −
GMm

2R

G9Mm

6R

4 GM
∴ V = √
7 R

Q8

Gm 2
F = 2
= mRω
(2R)

1 G
ω = √ 3
2 R

Q9

Option D is correct

2 4π 3
T = ⋅ r
GM

2 3

M =

G

r

 T

by putting values

23
M = 6 × 10

  #MathBoleTohMathonGo
www.mathongo.com
Kinetic
  Theory of Gases JEE Main 2021 (July) Chapter-wise Questions
Questions with Answer Keys MathonGo

Q1 (20 July 2021 Shift 1)

Consider a mixture of gas molecule of types A, B and C having masses m A


< mB < mC . The ratio of their

root mean square speeds at normal temperature and pressure is :

(1) v A
= vB = vC = 0

(2) 1

vA
>
1

vB
>
1

vC

(3) v A
= vB ≠ vC

(4) 1

vA
<
1

vB
<
1

vC

Q2 (20 July 2021 Shift 2)

The correct relation between the degrees of

freedom f and the ratio of specific heat γ is :

(1) f =
γ−1
2

(2) f =
γ+1
2

γ+1
(3) f =
2

(4) f =
γ+1
1

Q3 (22 July 2021 Shift 1)

What will be the average value of energy for a monoatomic gas in thermal equilibrium at temperature T ?

(1) 2

3
kB T

(2) k BT

(3)
3
kB T
2

(4)
1
kB T
2

Q4 (25 July 2021 Shift 1)

For a gas C P − CV = R in a state P and

CP − CV = 1.10R in a state Q, T and T are the

P Q

temperatures in two different states P and Q respectively. Then

  #MathBoleTohMathonGo
www.mathongo.com
Kinetic
  Theory of Gases JEE Main 2021 (July) Chapter-wise Questions
Questions with Answer Keys MathonGo

(1) T P = TQ

(2) T P < TQ

(3) T P = 0.9 TQ

(4) T P > TQ

Q5 (25 July 2021 Shift 2)

Two spherical soap bubbles of radii r and r in vacuum combine under isothermal conditions. The resulting
1 2

bubble has a radius equal to :


r1 r2
(1) r1 +r2

(2) √r 1 r2

(3) √r 2
1
+ r
2
2

r1 +r2
(4) 2

Q6 (25 July 2021 Shift 2)

A system consists of two types of gas molecules A and B having same number density 2 × 10 25
/m
3
. The
diameter of A and B are 10 A and 5 A respectively. They suffer collision at room temperature. The ratio of
average distance covered by the molecule A to that of B between two successive collision is ×10 −2

Q7 (27 July 2021 Shift 1)

The number of molecules in one litre of an ideal gas at 300 K and 2 atmospheric pressure with mean kinetic
energy 2 × 10 −9
 J per molecules is :

(1) 0.75 × 10 11

(2) 3 × 10 11

(3) 1.5 × 10 11

(4) 6 × 10 11

  #MathBoleTohMathonGo
www.mathongo.com
Kinetic
  Theory of Gases JEE Main 2021 (July) Chapter-wise Questions
Questions with Answer Keys MathonGo

Answer Key

Q1 (4) Q2 (1) Q3 (3) Q4 (4)

Q5 (3) Q6 (25) Q7 (3)

#MathBoleTohMathonGo
www.mathongo.com
Kinetic
  Theory of Gases JEE Main 2021 (July) Chapter-wise Questions
Hints and Solutions MathonGo

Q1

3RT
VRMS = √
M

mA < mB < mC

⇒ VA > VB > VC

1 1 1
⇒ < <
 VA  VB  VC

Q2

γ = 1 +
2

2
f =
γ−1

Q3

As per Equi-partition law :

Each degree of freedom contributes

2
kB T Average Energy

In monoatomic gas D.O.F. = 3

⇒ Average energy = 3 × 1

2
kB T =
3

2
kB T

Q4

CP − CV = R for ideal gas and gas behaves as ideal gas at high temperature so T P > TQ

Q5

no. of moles is conserved

n1 + n2 = n3

P1  V1 + P2  V2 = P3  V

4S 4 3 4S 4 3 4S 4 3
( πr ) + ( πr ) = ( πr )
r1 3 1 r2 3 2 r3 3 3

  #MathBoleTohMathonGo
www.mathongo.com
Kinetic
  Theory of Gases JEE Main 2021 (July) Chapter-wise Questions
Hints and Solutions MathonGo

r
2
1
+ r
2
2
= r
2
3

2 2
r3 = √r + r
1 2

Q6

∵ mean free path

λ =
1

√2πd n
2
λ1 d n2

λ2
=
2

 d n1
1

2
5 −2
= ( ) = 0.25 = 25 × 10
10

Q7

KE =
3

2
kT

PV =
N


RT

N =
PV

kT

11
= N = 1.5 × 10

#MathBoleTohMathonGo
www.mathongo.com
Laws
  of Motion JEE Main 2021 (July) Chapter-wise Questions
Questions with Answer Keys MathonGo

Q1 (20 July 2021 Shift 1)

The normal reaction 'N' for a vehicle of 800 kg mass, negotiating a turn on a 30 banked road at maximum

possible speed without skidding is ___ ×10 3


 kg m/s
2

(1) 10.2

(2) 7.2

(3) 12.4

(4) 6.96

Q2 (20 July 2021 Shift 1)

A steel block of 10 kg rests on a horizontal floor as shown. When three iron cylinders are placed on it as

shown, the block and cylinders go down with an acceleration 0.2 m/s . The normal reaction R by the floor if
2 ′

mass of the iron cylinders are equal and of 20 kg each, is N. [Take g = 10 m/s and 2
μs = 0.2]

(1) 716

(2) 686
(3) 714
(4) 684

Q3 (20 July 2021 Shift 2)

A body of mass ' m is launched up on a rough inclined plane making an angle of 30 with the horizontal. The
′ ∘

coefficient of friction between the

√x
body and plane is 5
if the time of ascent is half

of the time of descent. The value of x is

  #MathBoleTohMathonGo
www.mathongo.com
Laws
  of Motion JEE Main 2021 (July) Chapter-wise Questions
Questions with Answer Keys MathonGo

Q4 (22 July 2021 Shift 1)

A bullet of '4g' mass is fired from a gun of mass

4 kg . If the bullet moves with the muzzle speed of 50 ms , the impulse imparted to the gun and

−1

velocity of recoil of gun are :

(1) 0.4 kg ms −1
, 0.1 ms
−1

(2) 0.2 kg ms −1
, 0.05 ms
−1

(3) 0.2 kg ms −1
, 0.1 ms
−1

(4) 0.4 kg ms −1
, 0.05 ms
−1

Q5 (22 July 2021 Shift 1)

The motion of a mass on a spring, with spring constant K is as shown in figure.

The equation of motion is given by x(t) = Asin ωt+ B cos ωt with ω = √ K

Suppose that at time t = 0, the position of mass is

x(0) and velocity v(0), then its displacement can

also be represented as x(t) = C cos(ωt − ϕ), where C

and ϕ are:
2
2v(0) v(0)
(1) C = √ ω
2
2
+ x(0) , ϕ = tan
−1
(
x(0)ω
)

2
2v(0) x(0)ω
(2) C = √ ω
2
+ x(0)2 , ϕ = tan
−1
(
2v(0)
)

2
v(0) x(0)ω
(3) C = √ ω
2
2
+ x(0) , ϕ = tan
−1
(
v(0)
)

2
v(0) v(0)
(4) C = √ ω
2
2
+ x(0) , ϕ = tan
−1
(
x(0)ω
)

  #MathBoleTohMathonGo
www.mathongo.com
Laws
  of Motion JEE Main 2021 (July) Chapter-wise Questions
Questions with Answer Keys MathonGo

Q6 (25 July 2021 Shift 1)

Two billiard balls of equal mass 30 g strike a rigid

wall with same speed of 108kmph (as shown) but

at different angles. If the balls get reflected with

the same speed then the ratio of the magnitude of impulses imparted to ball 'a' and ball 'b' by the wall

along ' X ' direction is :

(1) 1 : 1

(2) √2 : 1
(3) 2: 1
(4) 1 : √2

Q7 (25 July 2021 Shift 2)

A force F→ = (40^i + 10^j)N acts on a body of mass 5 kg. If the body starts from rest, its position vector r at

time t = 10 s, will be :

(1) (100^i + 400^j)m


(2) (100^i + 100^j)m

(3) (400^i + 100^j)m


(4) (400^i + 400^j)m

  #MathBoleTohMathonGo
www.mathongo.com
Laws
  of Motion JEE Main 2021 (July) Chapter-wise Questions
Questions with Answer Keys MathonGo

Answer Key

Q1 (1) Q2 (2) Q3 (3) Q4 (2)

Q5 (4) Q6 (2) Q7 (3)

#MathBoleTohMathonGo
www.mathongo.com
Laws
  of Motion JEE Main 2021 (July) Chapter-wise Questions
Hints and Solutions MathonGo

Q1

At v max , f will be limiting in nature.

∴ Balancing force in vertical direction, N cos 30 ∘


− mg − μN cos 60

= 0

⇒ N [cos 30
∘ ∘
− μ cos 60 ] = mg

∴ N =
800×10

(0.87−0.1)
3
≈ 10.2 × 10  kg m/s
2

Hence option 1 .

Q2

Writing force equation in vertical direction

Mg − N = Ma

⇒ 70 g − N = 70 × 0.2

⇒ N = 70[ g − 0.2] = 70 × 9.8

∴ N = 686 Newton

Note : Since there is no compressive normal from

the sides, hence friction will not act.

Hence option 2.

  #MathBoleTohMathonGo
www.mathongo.com
Laws
  of Motion JEE Main 2021 (July) Chapter-wise Questions
Hints and Solutions MathonGo

Q3

ta =
1

2
td

․ √ ....(i)

2 s 1 2 s
= √
aa 2 ad

aa = g sin θ + μg cos θ

g √3
=
2
+
2
μg

ad = g sin θ − μg cos θ

g √3
=
2

2
μg

√3
using the above values of a and a and putting in (i) we will gate μ =
a d
5
eqution

Q4

By momentum conservation

4 × 10
−3
(50 − v) − 4v = 0

−3

4×10 ×50 −1
v = −3
≈ 0.05 ms
4+4×10

Impulse J = mv = 4 × .05 = 0.2kgms −1

Q5

x = A sin ωt + Bcosωt

v =
dx

dt
= Aω cos ωt − Bω sin ωt

At t = 0, x(0) = B

v(0) = Aω

x = A sin ωt + B sin(ωt + 90 )

  #MathBoleTohMathonGo
www.mathongo.com
Laws
  of Motion JEE Main 2021 (July) Chapter-wise Questions
Hints and Solutions MathonGo

2 2
Anet  = √A + B

B A
tan α = ⇒ cot α =
A B
2 2
⇒x = √A + B sin(ωt + α)

2 2
⇒ x = √A + B cos(ωt − (90 − α))

x = C cos(ωt − ϕ)

2 2
⇒ C = √A + B

2
[v(0)]
2
C = √ + [x(0)]
2
ω

ϕ = 90 − α

A
tan α = cos α =
B

v(0)
⇒ tan ϕ =
x(0)⋅ω

v(0)
−1
ϕ = tan ( )
x(0)ω

Q6

Impulse = change in momentum

−→
Ball (a) |Δp| = 2mu = J
1

−→
Ball (b) |Δp| = 2mu cos 45 ∘
= J2

J1 1
= ∘ = √2
 J2 cos 45

Q7



d v F 2
= a = = (8^
i + 2^
j )m/s
dt m



d r
= v = (8t^
i + 2t^
j )m/s
dt

→ 2

r = (8^
i + 2^
j)
t

2
 m

  #MathBoleTohMathonGo
www.mathongo.com
Laws
  of Motion JEE Main 2021 (July) Chapter-wise Questions
Hints and Solutions MathonGo

At t = 10sec


r = [(8^
i + 2^
j )50]m


⇒ r = (400^
i + 100^
j )m

#MathBoleTohMathonGo
www.mathongo.com
Magnetic
  Effects of Current JEE Main 2021 (July) Chapter-wise Questions
Questions with Answer Keys MathonGo

Q1 (20 July 2021 Shift 1)

A deuteron and an alpha particle having equal kinetic energy enter perpendicular into a magnetic field. Let r d

rd
and r be their respective radii of circular path. The value of
α
ra
is equal to:

(1) 1

√2

(2) √2

(3) 1

(4) 2

Q2 (25 July 2021 Shift 2)

Two ions having same mass have charges in the ratio 1 : 2. They are projected normally in uniform magnetic

field with their speeds in the ratio 2 : 3. The ratio of the radii of their circular trajectories is

(1) 1 : 4
(2) 4 : 3

(3) 3 : 1
(4) 2 : 3

Q3 (27 July 2021 Shift 2)

Figure A and B shown two long straight wires of circular cross-section (a and b with a < b ), carrying current
I which is uniformly distributed across the cross-section. The magnitude of magnetic field B varies with radius

r and can be represented as :

  #MathBoleTohMathonGo
www.mathongo.com
Magnetic
  Effects of Current JEE Main 2021 (July) Chapter-wise Questions
Questions with Answer Keys MathonGo

(1)

(2)

(3)

(4)

  #MathBoleTohMathonGo
www.mathongo.com
Magnetic
  Effects of Current JEE Main 2021 (July) Chapter-wise Questions
Questions with Answer Keys MathonGo

Answer Key

Q1 (2) Q2 (2) Q3 (3)

#MathBoleTohMathonGo
www.mathongo.com
Magnetic
  Effects of Current JEE Main 2021 (July) Chapter-wise Questions
Hints and Solutions MathonGo

Q1
mv √2mk
r = =
qB qB

rd md q
α 2 2
= √ = √ ( ) = √2
rα mα q 4 1
d

 Hence option (2). 

Q2
mv
1

R1 v1 q q v1

mv q1  B 2 2
R = ⇒ = mv = × = ×
qB R2 2 q v2 q v2
1 1
q2  B

2 2 4
= × ( ) =
1 3 3

Q3

Graph for wire of radius R :

As b > a

Ba > Bb

μ0 i
Ba =
2πa

μ0 i
Bb =
2πb

#MathBoleTohMathonGo
www.mathongo.com
Magnetic
  Properties of Matter JEE Main 2021 (July) Chapter-wise Questions
Questions with Answer Keys MathonGo

Q1 (20 July 2021 Shift 2)

At an angle of 30 to the magnetic meridian, the apparent dip is 45 . Find the true dip :
∘ ∘

(1) tan −1
√3

(2) tan −1 1

√3

(3) tan −1 2

√3

√3
(4) tan −1

Q2 (20 July 2021 Shift 2)

The magnetic susceptibility of a material of a rod is 499. Permeability in vacuum is 4π × 10 −7


H/m .

Absolute permeability of the material of the rod is:

(1) 4π × 10 −4
H/m

(2) 2π × 10 −4
H/m

(3) 3π × 10 −4
H/m

(4) π × 10 −4
H/m

Q3 (22 July 2021 Shift 1)

Statement I : The ferromagnetic property depends

on temperature. At high temperature, ferromagnet

becomes paramagnet.

Statement II : At high temperature, the domain

wall area of a ferromagnetic substance increases.

In the light of the above statements, choose the

most appropriate answer from the options given

below :

(1) Statement I is true but Statement II is false


(2) Both Statement I and Statement II are true

  #MathBoleTohMathonGo
www.mathongo.com
Magnetic
  Properties of Matter JEE Main 2021 (July) Chapter-wise Questions
Questions with Answer Keys MathonGo

(3) Both Statement I and Statement II are false


(4) Statement I is false but Statement II is true

Q4 (22 July 2021 Shift 1)

Choose the correct option:

(1) True dip is not mathematically related to apparent dip.


(2) True dip is less than apparent dip.
(3) True dip is always greater than the apparent dip.
(4) True dip is always equal to apparent dip.

Q5 (25 July 2021 Shift 1)

The value of aluminium susceptibility is 2.2 × 10 −5


The percentage increase in the magnetic field if space

within a current carrying toroid is filled with

aluminium is x
4
. Then the value of x is
10

Q6 (27 July 2021 Shift 1)

In a uniform magnetic field, the magnetic needle has a magnetic moment 9.85 × 10 −2
 A/m
2
and moment of
inertia 5 × 10 −6
 kg m
2
. If it performs 10 complete oscillations in 5 seconds then the magnitude of the
magnetic field is ___ mT

[Take π as
2
9.85]

  #MathBoleTohMathonGo
www.mathongo.com
Magnetic
  Properties of Matter JEE Main 2021 (July) Chapter-wise Questions
Questions with Answer Keys MathonGo

Answer Key

Q1 (4) Q2 (2) Q3 (1) Q4 (2)

Q5 (22) Q6 (8)

#MathBoleTohMathonGo
www.mathongo.com
Magnetic
  Properties of Matter JEE Main 2021 (July) Chapter-wise Questions
Hints and Solutions MathonGo
 
Q1

A tan δ = tan δ cos θ



= tan 45

cos 30

√3
tan δ = 1 ×
2

√3
−1
δ = tan ( )
2

Q2

μ = μ0 (1 + xm )

= 4π × 10
−7
× 500

−4
= 2π × 10 H/m

Q3

As temperature increases, domains disintegrate so ferromagnetism decreases and above curie

temperature it become paramagnet.

Q4

#MathBoleTohMathonGo
www.mathongo.com
Magnetic
  Properties of Matter JEE Main 2021 (July) Chapter-wise Questions
Hints and Solutions MathonGo
 
If apparent dip circle is at an angle α with true dip circle then

Q5

Sol. B = μ ⋅ (H + I)

B = μ ⋅ H (1 +
I

H
)

B = B0 (1 + x)

B − B0 = B0 X

B−B0

 B0
= x

B−Bb

B0
× 100 = 100x

−3 22
= 2.2 × 10 = 4
10

Q6

I
T = 2π√
MB

−4
B = 80 × 10 = 8mT

#MathBoleTohMathonGo
www.mathongo.com
Magnetic
  Properties of Matter JEE Main 2021 (July) Chapter-wise Questions
Hints and Solutions MathonGo

#MathBoleTohMathonGo
www.mathongo.com
Mathematics
  in Physics JEE Main 2021 (July) Chapter-wise Questions
Questions with Answer Keys MathonGo

Q1 (20 July 2021 Shift 1)


→ → → → → → → →
If A and B are two vectors satisfying the relation A ⋅ B = | A × B |. Then the value of | A − B| will be

(1) √A 2
+ B
2

(2) √A 2
+ B
2
+ √2AB

(3) √A 2
+ B
2
+ 2AB

(4) √A 2
+ B
2
− √2AB

Q2 (20 July 2021 Shift 2)


→ →
Two vectors P and Q have equal magnitudes. If

→ →
the magnitude of P + Q is n times the magnitude

→ → → →
of P − Q , then angle between P and Q is :
n−1
(1) sin −1
(
n+1
)

n−1
(2) cos −1
(
n+1
)

2
n −1
(3) sin −1
( 2
n +1
)

2
n −1
(4) cos −1
( 2
n +1
)

Q3 (22 July 2021 Shift 1)



What will be the projection of vector A = ^
i + ^
^
j + k


on vector B = ^
i + ^
j ?

(1) √2(^i + ^j + k)
^

(2) 2(^i + ^j + k)
^

(3) √2(^i + ^j)


(4) (^i + ^j)

Q4 (22 July 2021 Shift 1)


→ → →
Three particles P, Q and R are moving along the vectors A = ^
i + ^
j, B = ^ ^
j + k and C = −^
i + ^
j respectively.

They strike on a point and start to

→ →
move in different directions. Now particle P is moving normal to the plane which contains vector A and B . Similarly

  #MathBoleTohMathonGo
www.mathongo.com
Mathematics
  in Physics JEE Main 2021 (July) Chapter-wise Questions
Questions with Answer Keys MathonGo
→ →
particle Q is moving normal to the plane which contains vector A and C . The angle between the direction of motion of

and Q is cos Then the value of x is _____.


−1 1
P ( ).
√x

Q5 (22 July 2021 Shift 1)

Three students S 1,  S2 and S perform an experiment for determining the acceleration due to gravity (g) using a simple
3

pendulum. They use different lengths of pendulum and record time for different number of oscillations. The observations

are as shown in the table.

Student No. Length of pendulum (cm) No. of oscilations (n) Total time for n oscillations Time period (s)
1. 64.0 8 128.0 16.0

2. 64.0 4 64.0 16.0

3. 20.0 4 36.0 9.0

(Least count of length = 0.1 m least count for time = 0.1 s ) If E 1, E2 and E are the percentage errors in 'g' for students
3

1,2 and 3 respectively, then the minimum percentage error is obtained by student no. ____.

Q6 (25 July 2021 Shift 1)

Match List I with List II.

(1) (a) → (iv), (b) → (i), (c) → (iii), (d) → (ii)


(2) (a) → (iv), (b) → (iii) , (c) → (i), (d) → (ii)
(3) (a) → (iii), (b) → (ii), (c) → (iv), (d) → (i)

#MathBoleTohMathonGo
www.mathongo.com
Mathematics
  in Physics JEE Main 2021 (July) Chapter-wise Questions
Questions with Answer Keys MathonGo

(4) (a) → (i), (b) → (iv), (c) → (ii), (d) → (iii)

Q7 (25 July 2021 Shift 2)


→ →
→ and Y→ have equal magnitude. The magnitude of ( X
Two vectors X − Y) is n times the magnitude of

→¯)
(X + Y → and Y→ is :
. The angle between X
2
−n −1
(1) cos −1
(
n −1
2
)

2
n −1
(2) cos −1
(
−n −1
2
)

2
n +1
(3) cos −1
(
−n −1
2
)

2
n +1
(4) cos −1
( 2
n −1
)

Q8 (27 July 2021 Shift 1)



−→ −
−→ −
−→
Assertion A : If A, B, C, D are four points on a semi-circular arc with centre at 'O' such that |AB | = |BC | = |CD |, then


−→ −
−→ −
−→ −
−→ −
−→ −
−→
AB + AC + AD = 4AO + OB + OC


−→ −
−→ −
−→ −
−→ −
−→
Reason R : Polygon law of vector addition yields AB + BC + CD + AD = 2AO

In the light of the above statements, choose the most appropriate answer from the options given below :

(1) A is correct but R is not correct.


(2) A is not correct but R is correct.

(3) Both A and R are correct and R is the correct explanation of A.


(4) Both A and R are correct but R is not the correct explanation of A.

Q9 (27 July 2021 Shift 2)


3

A physical quantity 'y' is represented by the formula y = m 2


r
−4 x −
 g l 2

If the percentage errors found in y, m, r, l and g are 18, 1, 0.5, 4 and p respectively, then find the value of x and p.

(1) 5 and ±2
(2) 4 and ±3
(3) 16

3
and ± 3

  #MathBoleTohMathonGo
www.mathongo.com
Mathematics
  in Physics JEE Main 2021 (July) Chapter-wise Questions
Questions with Answer Keys MathonGo

(4) 8 and ±2

  #MathBoleTohMathonGo
www.mathongo.com
Mathematics
  in Physics JEE Main 2021 (July) Chapter-wise Questions
Questions with Answer Keys MathonGo

Answer Key

Q1 (4) Q2 (4) Q3 (4) Q4 (3)

Q5 (1) Q6 (2) Q7 (2) Q8 (4)

Q9 (3)

#MathBoleTohMathonGo
www.mathongo.com
Mathematics
  in Physics JEE Main 2021 (July) Chapter-wise Questions
Hints and Solutions MathonGo

Q1
→ → → →
A ⋅ B = |A × B|

AB cos θ = AB sin θ ⇒ θ = 45

→ →
2 2
| A − B | = √A + B − 2AB cos 45

= √A
2
+ B
2
− √2AB

Hence option (4).

Q2
→ →
| P | = |Q| = x ...(i)

→ → → →
| P + Q | = n| P − Q |

P
2
+ Q
2
+ 2PQ cos θ = n
2
(P
2
+ Q
2
− 2PQ cos θ)

Using (i) in above equation

2
n −1
cos θ =
1+n
2

2
n −1
−1
θ = cos ( 2
)
n +1

Q3
→ → → →
^ = A(
(A cos θ)B
A⋅B

AB
^ =
)B
A⋅B

B
^
B

^ ^
2 i +j
= ( ) = ^
i + ^
j
√2 √2

Q4
→ →
^ ^ ^
i − j +k
Direction of P^

A×B
v 1 = ± → →
= ±
√3
|A×B|

→ →
^
Direction of Q^
v 2 = ± →
A×C

= ±
2k

2
^
= ±k

|A×C|

Angle between v^ and v^


1 2

^1 ⋅ v
^2 ±1/√3
v

^ ∥v
|v ^ |
=
(1)(1)
= ±
1

1 2 √3

⇒ x = 3

Q5

  #MathBoleTohMathonGo
www.mathongo.com
Mathematics
  in Physics JEE Main 2021 (July) Chapter-wise Questions
Hints and Solutions MathonGo
2

ℓ 4π ℓ
T = 2π√ ⇒ g =
g 2
T

Δg

g
=
Δℓ


+
2ΔT

 least count of time (ΔT0 )


ΔT =
 number of oscillations(n) 

Δg 2ΔT0

g
=
Δℓ


+
nT

As Δℓ and ΔT are same for all observations so

Δg

 is minimum for highest value of ℓ,n and T


Q

⇒ Minimum percentage error in g is for student number-1

Q6
→ → →
(a) C = A + B

Option (iv)

→ → → → →
(b) A = B + C = C + B

Option (iii)

→ → →
(c) B = A + C

Option (i)

→ → →
(d) A + B + C = 0

Option (ii)

Q7

 Given X = Y

√X
2
+ Y
2
− 2 × Y cos θ

2 2
= n√ X + Y + 2 × Y cos θ

Square both sides 2X 2


(1 − cos θ) = n
2 2
⋅ 2X (1 + cos θ)

1 − cos θ = n
2
+ n
2
cos θ

2
1−n
cos θ =
1+n
2

2
n −1
−1
θ = cos [ 2
]
−n −1

Q8

Polygon law is applicable in both but the equation given in the reason is not useful in explaining the assertion.

  #MathBoleTohMathonGo
www.mathongo.com
Mathematics
  in Physics JEE Main 2021 (July) Chapter-wise Questions
Hints and Solutions MathonGo

Q9
Δy xΔg

2Δm 4Δr 3 Δℓ
= + + +
y m r g 2 ℓ

18 = 2(1) + 4(0.5) + xp +
3

2
(4)

8 = xp

By checking from options. x = 16

3
,p = ±
3

#MathBoleTohMathonGo
www.mathongo.com
Mechanical
  Properties of Fluids JEE Main 2021 (July) Chapter-wise Questions
Questions with Answer Keys MathonGo

Q1 (20 July 2021 Shift 2)

Two small drops of mercury each of radius R coalesce to form a single large drop. The ratio of

total surface energy before and after the change is:


1

(1) 2 3
: 1

(2) 1 : 2 3

(3) 2 : 1

(4) 1 : 2

Q2 (27 July 2021 Shift 1)

A light cylindrical vessel is kept on a horizontal surface. Area of base is A. A hole of cross- sectional area 'a' is

made just at its bottom side. The minimum coefficient of friction necessary to prevent sliding the vessel due to

the impact force of the emerging liquid is $(a<

(1) A

2a

(2) None of these


(3) 2a

(4) a

Q3 (27 July 2021 Shift 2)

A raindrop with radius R = 0.2 mm falls from a

cloud at a height h = 2000 m above the ground.

Assume that the drop is spherical throughout its

fall and the force of buoyance may be neglected,

then the terminal speed attained by the raindrop is :

  #MathBoleTohMathonGo
www.mathongo.com
Mechanical
  Properties of Fluids JEE Main 2021 (July) Chapter-wise Questions
Questions with Answer Keys MathonGo

[Density of water f w = 1000 kg m


−3
and Density of

air f a = 1.2 kg m
−3
,  g = 10 m/s
2

Coefficient of viscosity of air = 1.8 × 10 −5


Nsm
−2
]

(1) 250.6 ms −1

(2) 43.56 ms −1

(3) 4.94 ms −1

(4) 14.4 ms −1

  #MathBoleTohMathonGo
www.mathongo.com
Mechanical
  Properties of Fluids JEE Main 2021 (July) Chapter-wise Questions
Questions with Answer Keys MathonGo

Answer Key

Q1 (1) Q2 (3) Q3 (3)

#MathBoleTohMathonGo
www.mathongo.com
Mechanical
  Properties of Fluids JEE Main 2021 (July) Chapter-wise Questions
Hints and Solutions MathonGo

Q1

3
πR
3
+
4

3
πR
3
=
4

3
πR
′3

R

= 23 R ...(i)

Ai = 2 [4πR ]
2

Af = 4πR
′2

2
Ui Ai 2R 1/3
= = = 2
Uf 2/3 2
Af 2 R

Q2

For no sliding

f ≥ ρav
2

μmg ≥ ρav
2

μρAhg ≥ ρa2gh

μ ≥
2a

Option (3)

Q3

  #MathBoleTohMathonGo
www.mathongo.com
Mechanical
  Properties of Fluids JEE Main 2021 (July) Chapter-wise Questions
Hints and Solutions MathonGo

At terminal speed

a = 0

 Fnet  = 0

mg = Fv = 6πηRv

mg
v =
6πRv
4π 3
ρw R  g
3
v =
6πηR
2
2ρw R  g
=

400
=  m/s
81

= 4.94 m/s

#MathBoleTohMathonGo
www.mathongo.com
Mechanical
  Properties of Solids JEE Main 2021 (July) Chapter-wise Questions
Questions with Answer Keys MathonGo

Q1 (20 July 2021 Shift 1)

The value of tension in a long thin metal wire has been changed from T to T . The lengths of the metal wire
1 2

at two different values of tension T and T are ℓ and ℓ respectively. The actual length of the metal wire is :
1 2 1 2

T1 ℓ2 −T2 ℓ1
(1)  T1 −T2

T1 ℓ1 −T2 ℓ2
(2)  T1 −T2

ℓ1 +ℓ2
(3) 2

(4) √T 1  T2 ℓ1 ℓ2

Q2 (20 July 2021 Shift 2)

The length of a metal wire is ℓ , when the tension

in it is T and is ℓ when the tension is T . The

1 2 2

natural length of the wire is :

(1) √ℓ 1 ℓ2

ℓ1  T2 −ℓ2  T1


(2)  T2 −T1

ℓ1  T2 +ℓ2  T1


(3)  T2 +T1

ℓ1 +ℓ2
(4) 2

Q3 (22 July 2021 Shift 1)

8. The area of cross-section of a railway track is 0.01 m . The temperature variation is 10


2 ∘
C . Coefficient of
linear expansion of material of track is 10 −5 ∘
/ C . The energy stored per meter in the track is ____ J/m

(Young's modulus of material of track is 10 11


Nm
−2
)

Q4 (25 July 2021 Shift 1)

Two wires of same length and radius are joined

end to end and loaded. The Young's modulii of the materials of the two wires are Y and Y . The

1 2

combination behaves as a single wire then its

Young's modulus is :

  #MathBoleTohMathonGo
www.mathongo.com
Mechanical
  Properties of Solids JEE Main 2021 (July) Chapter-wise Questions
Questions with Answer Keys MathonGo
2Y1 Y2
(1) Y = 3(Y1 +Y2 )

2Y1 Y2
(2) Y = Y1 +Y2

Y1 Y2
(3) Y = 2(Y1 +Y2 )

Y1 Y2
(4) Y =
Y1 +Y2

Q5 (27 July 2021 Shift 1)

A stone of mass 20 g is projected from a rubber catapult of length 0.1 m and area of cross section 10 −6
 m
2

stretched by an amount 0.04 m. The velocity of the projected stone is ___ m/s

(Young's modulus of rubber = 0.5 × 10 9


 N/m
2
)

  #MathBoleTohMathonGo
www.mathongo.com
Mechanical
  Properties of Solids JEE Main 2021 (July) Chapter-wise Questions
Questions with Answer Keys MathonGo

Answer Key

Q1 (1) Q2 (2) Q3 (5) Q4 (2)

Q5 (20)

#MathBoleTohMathonGo
www.mathongo.com
Mechanical
  Properties of Solids JEE Main 2021 (July) Chapter-wise Questions
Hints and Solutions MathonGo

Q1

FL
Y =
AΔL

T 1 ℓ0 T 2 ℓ0
⇒ Y =
 A(ℓ1 −ℓ0 )
=
 A(ℓ2 −ℓ0 )

T1 (ℓ2 −ℓ0 )
1 =
T2 (ℓ1 −ℓ0 )

T2 ℓ 1 − T2 ℓ 0 = T1 ℓ 2 − T1 ℓ 0

( T1 − T2 ) ℓ0 = T1 ℓ2 − T2 ℓ1

T1 ℓ2 −T2 ℓ1
ℓ0 = ( )
 T1 −T2

Q2

T1 = k (ℓ1 − ℓ0 )

T2 = k (ℓ2 − ℓ0 )

T1 ℓ1 −ℓ0

 T2
=
ℓ2 −ℓ0

T1 ℓ2 −T2 ℓ1
= ℓ0
 T1 −T2

Q3

Elastic energy = Y

2
( strain ) ×
2
Area × length

⇒ Elastic energy per unit length = Y

2
( strain ) ×
2
Area

( strain =
Δℓ


= αΔT = 10
−5
× 10 = 10
−4
)

11
10 2
−4 −2
= × (10 ) × 10 = 5 J/m
2

Q4

In series combination Δ1 = ℓ 1 + ℓ2

F/A
Y =
Δℓ/ℓ
⇒ Δℓ =
Fℓ

AY


⇒ Δℓ ∝
Y

Equivalent length of rod after joing is = 2ℓ

As, lengths are same and force is also same in

series

Δℓ = Δℓ1 + Δℓ2

  #MathBoleTohMathonGo
www.mathongo.com
Mechanical
  Properties of Solids JEE Main 2021 (July) Chapter-wise Questions
Hints and Solutions MathonGo
ℓeq

ℓ ℓ 2ℓ ℓ ℓ
= + ⇒ = +
Yeq Y1 Y2 Y Y1 Y2

2Y1 Y2
Y =
Y1 +Y2

Q5

By energy conservation

2

YA

L
⋅ x
2
=
1

2
mv
2

9 −6 2
0.5×10 ×10 ×(0.04)

0.1
=
20

1000
v
2

∴ v
2
= 400

v = 20 m/s

#MathBoleTohMathonGo
www.mathongo.com
Motion
  In One Dimension JEE Main 2021 (July) Chapter-wise Questions
Questions with Answer Keys MathonGo

Q1 (20 July 2021 Shift 2)

A body at rest is moved along a horizontal straight line by a machine delivering a constant power. The distance

moved by the body in time ' t is proportional to:


(1) t 2

(2) t 2

(3) t 4

(4) t 4

Q2 (25 July 2021 Shift 1)

Water droplets are coming from an open tap at particular rate. The spacing between a droplet

observed at 4 th 
second after its fall to the next

droplet is 34.3 m. At what rate the droplets are coming from the tap ? (Take g = 9.8 m/s ) 2

(1) 3 drops /2 seconds


(2) 2 drops / second

(3) 1 drop / second


(4) 1 drop /7 seconds

Q3 (25 July 2021 Shift 2)

The relation between time t and distance x for a moving body is given as t = mx 2
+ nx , where m land n are
constants. The retardation of the motion

(1) 2mv 3

(2) 2mnv 3

(3) 2nv 3

(4) 2n 2
v
3

Q4 (25 July 2021 Shift 2)

  #MathBoleTohMathonGo
www.mathongo.com
Motion
  In One Dimension JEE Main 2021 (July) Chapter-wise Questions
Questions with Answer Keys MathonGo

A balloon was moving upwards with a uniform velocity of 10 m/s. An object of finite mass is dropped from
the balloon when it was at a height of 75 m from the ground level. The height of the balloon from the ground
when object strikes the ground was around

(takes the value of g as 10 m/s ) 2

(1) 300 m

(2) 200 m
(3) 125 m

(4) 250 m

Q5 (25 July 2021 Shift 2)

The instantaneous velocity of a particle moving in a straight line is given as v = αt + βt , where α and β are
2

constants. The distance travelled by the particle between 1 s and 2 s is :

(1) 3α + 7β
(2) 3

2
α +
7

3
β

β
(3) α

2
+
3

(4)
3 7
α + β
2 2

Q6 (27 July 2021 Shift 1)

A ball is thrown up with a certain velocity so that it reaches a height 'h'. Find the ratio of the two different
times of the ball reaching in both the directions.
h

√2−1
(1) √2+1

(2)
1

√3−√2
(3) √3+√2

√3−1
(4)
√3+1

Q7 (27 July 2021 Shift 2)

  #MathBoleTohMathonGo
www.mathongo.com
Motion
  In One Dimension JEE Main 2021 (July) Chapter-wise Questions
Questions with Answer Keys MathonGo

A particle of mass M originally at rest is subjected

to a force whose direction is constant but

magnitude varies with time according to the

relation

2
t − T
F = F0 [1 − ( ) ]
T

Where F and T are constants. The force acts only

for the time interval 2 T. The velocity v of the

particle after time 2 T is :

(1) 2 F 0
 T/M

(2) F 0
 T/2M

(3) 4 F 0
 T/3M

(4) F 0
 T/3M

  #MathBoleTohMathonGo
www.mathongo.com
Motion
  In One Dimension JEE Main 2021 (July) Chapter-wise Questions
Questions with Answer Keys MathonGo

Answer Key

Q1 (1) Q2 (3) Q3 (1) Q4 (3)

Q5 (2) Q6 (3) Q7 (3)

#MathBoleTohMathonGo
www.mathongo.com
Motion
  In One Dimension JEE Main 2021 (July) Chapter-wise Questions
Hints and Solutions MathonGo

Q1

P = constant

1 2
mv = Pt
2

⇒ v ∝ √t

dx

dt
= C√ t C = constant

by integration.

3/2
x ∝ t

Q2

In 4sec.1 drop will travel ⇒


s  1

2
× (9.8) × (4)
2
= 78.4 m

∴ 2
nd 
drop would have travelled ⇒ 78.4 − 34.3 = 44.1 m

Time for 2 nd 


drop 1

2
(9.8)t
2
= 44.1

t = 3 sec

each drop have time gap of 1sec

∴ 1 drop per sec

Q3

t = mx
2
+ nx

v
=
dt

dx
= 2mx + n

1
v =
2mx+n

dv 2 m dx
= − 2
( )
dt (2 mx+n) dt

3
a = −(2 m)v

Q4

  #MathBoleTohMathonGo
www.mathongo.com
Motion
  In One Dimension JEE Main 2021 (July) Chapter-wise Questions
Hints and Solutions MathonGo

Object is projected as shown so as per motion under gravity S = ut +

1 2
at
2

1 2
−75 = +10t + (−10)t ⇒ t = 5sec
2

Object takes t = 5 s to fall on ground Height of balloon from ground H = 75 + ut

= 75 + 10 × 5 = 125 m

Q5

V = αt + βt
2

ds 2
= αt + βt
dt

s2 2

S1
ds = ∫
1
(αt + βt ) dt
2

3 2
2 βt
S2 − S1 = [
αt

2
+
3
]

α[4−1] β[8−1]
As particle is not changing direction So distance = displacement. Distance = [ 2
+
3
]

3α 7β
= +
2 3

Q6

u = √2gh

Now, S =

h
a = −g
3

S = ut +
1

2
at
2

3
= √2ght +
1

2
(−g)t
2

g
t
2
(
2
) − √2ght +
h

3
= 0

From quadratic equation

4 g
h
√2gh±√2gh−

2 3

t1 , t2 =
 g

  #MathBoleTohMathonGo
www.mathongo.com
Motion
  In One Dimension JEE Main 2021 (July) Chapter-wise Questions
Hints and Solutions MathonGo
4gh
√2gh−√
t1

=
t2 4gh
√2gh+√
3

√3−√2
=
√3+√2

Q7

t = 0, u = 0

Fo Fo
a =
M

2
(t − T)
2
=
dv

dt

MT

v Fo Fo

0
dv = ∫
2 T

t=0
(
M

2
2
(t − T) ) dt

MT
2 T 3 2 T
F0 Fo
V = [
M
t] −
2
[
t

3
2
− t  T + T t]
2

MT
o 0

4 F0  T
 V =
3M

#MathBoleTohMathonGo
www.mathongo.com
Motion
  In Two Dimensions JEE Main 2021 (July) Chapter-wise Questions
Questions with Answer Keys MathonGo

Q1 (20 July 2021 Shift 1)

A butterfly is flying with a velocity 4√2 m/s in North-East direction. Wind is slowly blowing at 1 m/s from

North to South. The resultant displacement of the butterfly in 3 seconds is :

(1) 3 m

(2) 20 m

(3) 12√2 m

(4) 15 m

Q2 (20 July 2021 Shift 1)

In a spring gun having spring constant 100 N/m a small ball 'B' of mass 100 g is put in its barrel (as shown in

figure) by compressing the spring through 0.05 m. There should be a box placed at a distance 'd' on the ground
so that the ball falls in it. If the ball leaves the gun horizontally at a height of 2 m above the ground. The value

of d is ___ m.

(g = 10 m/s )
2

Q3 (20 July 2021 Shift 2)

A boy reaches the airport and finds that the escalator is not working. He walks up the stationary escalator in

time t . If he remains

stationary on a moving escalator then the escalator takes him up in time t . The time taken by him to walk up
2

on the moving escalator will be :


t1 t2
(1) t2 −t1

t1 +t2
(2) 2

  #MathBoleTohMathonGo
www.mathongo.com
Motion
  In Two Dimensions JEE Main 2021 (July) Chapter-wise Questions
Questions with Answer Keys MathonGo
t1 t2
(3) t2 +t1

(4) t 2 − t1

Q4 (27 July 2021 Shift 2)

A swimmer wants to cross a river from point A to point B. Line AB makes an angle of 30 with the flow of

river. Magnitude of velocity of the swimmer is same as that of the river. The angle θ with the line AB should

be , so that the swimmer reaches point B.

Q5 (27 July 2021 Shift 2)

A small block slides down from the top of

hemisphere of radius R = 3 m as shown in the

figure. The height 'h' at which the block will lose

contact with the surface of the sphere is m.

(Assume there is no friction between the block and

the hemisphere)

Q6 (27 July 2021 Shift 2)

The water is filled upto height of 12 m in a tank having vertical sidewalls. A hole is made in one of the walls
at a depth 'h' below the water level. The

  #MathBoleTohMathonGo
www.mathongo.com
Motion
  In Two Dimensions JEE Main 2021 (July) Chapter-wise Questions
Questions with Answer Keys MathonGo

value of 'h' for which the emerging stream of water strikes the ground at the maximum range is m.

  #MathBoleTohMathonGo
www.mathongo.com
Motion
  In Two Dimensions JEE Main 2021 (July) Chapter-wise Questions
Questions with Answer Keys MathonGo

Answer Key

Q1 (4) Q2 (1) Q3 (3) Q4 (30)

Q5 (2) Q6 (6)

#MathBoleTohMathonGo
www.mathongo.com
Motion
  In Two Dimensions JEE Main 2021 (July) Chapter-wise Questions
Hints and Solutions MathonGo

Q1


V BW = 4√2 cos 45^
i + 4√2 sin 45^
j

= 4^
i + 4^
j


V w = −^
j

→ → →
V B = V BW + V w = 4^
i + 3^
j

→ →
S B
= V B × t = (4^
i + 3^
j ) × 3 = 12^
i + 9^
j


∣ ∣
2 2
∣ S B ∣ = √(12) + (9) = 15 m
∣ ∣

Q2

2
kx
2
=
1

2
mv
2

Kx
2
= mv
2

k 100
v = x√ = 0.05√ = 0.05 × 10√10
m 0.1

v = 0.5√10

  #MathBoleTohMathonGo
www.mathongo.com
Motion
  In Two Dimensions JEE Main 2021 (July) Chapter-wise Questions
Hints and Solutions MathonGo

From h =

1 2
gt
2

t = √
2 h

 g
= √
2×2

10
=
2

√10

2
∴ d = vt = 0.5√10 × = lm
√10

Q3

L = Length of escalator

Vb/esc =
L

t1

When only escalator is moving.

Vesc =
L

t2

when both are moving

Vb/g = Vb/esc + Vesc 

L L L t1 t2
Vb/g = + ⇒ [t = = ]
t1 t2 Vb/g t1 +t2

Q4

Both velocity vectors are of same magnitude therefore resultant would pass exactly midway

through them θ = 30 ∘

Q5

mg cos θ =
mv

  #MathBoleTohMathonGo
www.mathongo.com
Motion
  In Two Dimensions JEE Main 2021 (July) Chapter-wise Questions
Hints and Solutions MathonGo

cos θ =
h

Energy conservation

mg{R − h} =
1

2
mv
2

2mg{R−h}
from (1) & (2) ⇒ mg { h

R
} =
R

2R
h = = 2 m
3

Q6

(12−h)×2
R = √2gh × √  g

√4 h(12 − h) = R

For maximum R

dR

dh
= 0

⇒ h = 6 m

#MathBoleTohMathonGo
www.mathongo.com
Nuclear
  Physics JEE Main 2021 (July) Chapter-wise Questions
Questions with Answer Keys MathonGo

Q1 (20 July 2021 Shift 1)

A radioactive material decays by simultaneous emissions of two particles with half lives of 1400 years and
700 years respectively. What will be the time after the which one third of the material remains ? (Take

ln 3 = 1.1 )

(1) 1110 years

(2) 700 years

(3) 340 years

(4) 740 years

Q2 (20 July 2021 Shift 1)

A nucleus of mass M emits γ -ray photon of frequency ' v . The loss of internal energy by the nucleus is:

[Take 'c' as the speed of electromagnetic wave]

(1) hv
(2) 0

(3) hv [1 − hv

2
]
2Mc

(4) hv [1 + hv

2
]
2Mc

Q3 (20 July 2021 Shift 2)

For a certain radioactive process the graph between

In R and t(sec) is obtained as shown in the figure.

Then the value of half life for the unknown

  #MathBoleTohMathonGo
www.mathongo.com
Nuclear
  Physics JEE Main 2021 (July) Chapter-wise Questions
Questions with Answer Keys MathonGo

radioactive material is approximately :

(1) 9.15sec
(2) 6.93sec

(3) 2.62sec
(4) 4.62sec

Q4 (20 July 2021 Shift 2)


th 

A radioactive substance decays to ( 1

16
) of its

initial activity in 80 days. The half life of the radioactive substance expressed in days is ____.

Q5 (22 July 2021 Shift 1)

A nucleus with mass number 184 initially at rest emits an α -particle. If the Q value of the reaction is 5.5MeV,

calculate the kinetic energy of the α− particle.

(1) 5.0MeV
(2) 5.5MeV
(3) 0.12MeV
(4) 5.38MeV

Q6 (25 July 2021 Shift 1)

  #MathBoleTohMathonGo
www.mathongo.com
Nuclear
  Physics JEE Main 2021 (July) Chapter-wise Questions
Questions with Answer Keys MathonGo

Some nuclei of a radioactive material are undergoing radioactive decay. The time gap between the instances
when a quarter of the nuclei have decayed and when half of the nuclei have decayed is given as:

(where λ is the decay constant)

(1)
1 ln 2

2 λ

(2)
ln 2

(3) 2ln2

λ
d
ln

(4) 2
2

Q7 (25 July 2021 Shift 1)

The half-life of 198


Au is 3 days. If atomic weight of 198
Au is 198 g/mol then the activity of 2mg of 198
Au is
[in disintegration/second] :

(1) 2.67 × 10 12

(2) 6.06 × 10 18

(3) 32.36 × 10 12

(4) 16.18 × 10 12

Q8 (25 July 2021 Shift 2)

From the given data, the amount of energy required to break the nucleus of aluminium 27

13
Al is x × 10 −3
 J

Mass of neutron = 1.00866u

Mass of proton = 1.00726u

Mass of Aluminium nucleus = 27.18846u

(Assume 1 u corresponds to x J of energy) (Round off to the nearest integer)

Q9 (25 July 2021 Shift 2)

The nuclear activity of a radioactive element

th 

becomes ( of its initial value in 30 years. The

1
)
8

half-life of radioactive element is years.

  #MathBoleTohMathonGo
www.mathongo.com
Nuclear
  Physics JEE Main 2021 (July) Chapter-wise Questions
Questions with Answer Keys MathonGo

Q10 (27 July 2021 Shift 1)

If 'f' denotes the ratio of the number of nuclei decayed (N d


) to the number of nuclei at t = 0 ( N 0) then for a
collection of radioactive nuclei, the rate of change of ' f with respect to time is given as:

[λ is the radioactive decay constant ]

(1) −λ (1 − e −λt
)

(2) λ (1 − e −λt
)

(3) λe −λt

(4) −λe −λd

Q11 (27 July 2021 Shift 1)

A radioactive sample has an average life of 30 ms and is decaying. A capacitor of capacitance 200μF is first
charged and later connected with resistor ′
R

. If the ratio of charge on capacitor to the activity of radioactive

sample is fixed with respect to time then the value of 'R' should be ___ Ω.

Q12 (27 July 2021 Shift 2)

Consider the following statements:

A. Atoms of each element emit characteristics

spectrum.

B. According to Bohr's Postulate, an electron in a hydrogen atom, revolves in a certain stationary orbit.

C. The density of nuclear matter depends on the size of the nucleus.

D. A free neutron is stable but a free proton decay is possible.

E. Radioactivity is an indication of the instability of nuclei. Choose the correct answer from the options given

below :

(1) A, B, C, D and E
(2) A, B and E only

(3) B and D only

(4) A, C and E only

  #MathBoleTohMathonGo
www.mathongo.com
Nuclear
  Physics JEE Main 2021 (July) Chapter-wise Questions
Questions with Answer Keys MathonGo

Answer Key

Q1 (4) Q2 (4) Q3 (4) Q4 (20)

Q5 (4) Q6 (4) Q7 (4) Q8 (27)

Q9 (10) Q10 (3) Q11 (150) Q12 (2)

#MathBoleTohMathonGo
www.mathongo.com
Nuclear
  Physics JEE Main 2021 (July) Chapter-wise Questions
Hints and Solutions MathonGo

Q1

Given λ 1 =
ln 2

700
/ year, λ 2 =
ln 2

1400
/ year

∴ λmet = λ1 + λ2 = ℓn2 [
1

700
+
1400
1
]

=
3ℓn2

1400
/ year

Now, Let initial no. of radioactive nuclei be

No.

N0

3
= N0 e
−λmat 

⇒ ℓn
1

3
= −λnet t

⇒ 1.1 =
3×0.693

1400
t ⇒ t ≈ 740 years

Hence option 4

Q2

Energy of γ ray [E γ] = hv Momentum of γ ray [P γ] =


h

λ
=
hv

→ →
Total momentum is conserved. P y
+ P Nu
= 0


Where P Nu
= Momentum of decayed nuclei ⇒ P γ
= PNu


hv

C
= PNu

⇒ K. E . of nuclei

2 2
(PNu )
=
1

2
Mv
2
=
2M
=
1

2M
[
hv

C
]

Loss in internal energy = E γ + K ⋅ ENu

1 hv
= hv + [ ]
2M C

hv
= hv[1 + 2
]
2MC

Q3

  #MathBoleTohMathonGo
www.mathongo.com
Nuclear
  Physics JEE Main 2021 (July) Chapter-wise Questions
Hints and Solutions MathonGo

R = R0 e
−λt

ℓnR = ℓnR0 − λt

−λ is slope of straight line

3
λ =
20

ln 2
t1/2 = = 4.62
λ

Q4

4 × t1/2 = 80

t1/2 = 20 days

Q5

2
1 2
1 4v
(4m)v + (180m)( ) = 5.5MeV
2 2 180

2
1 4
2
⇒ 4mv [1 + 45( ) ] = 5.5MeV
2 180

5.5
⇒ K. Eα = MeV
2
4
1 + 45 ⋅ ( )
180

K. Eα = 5.38MeV

Q6

  #MathBoleTohMathonGo
www.mathongo.com
Nuclear
  Physics JEE Main 2021 (July) Chapter-wise Questions
Hints and Solutions MathonGo
3 N0

4
= N0 e
−it1

N0

2
= N0 e
−λ2

ln(3/4) = −λt1

ln(1/2) = −λt2 … . (i)

ln(3/4) − ln(1/2) = λ (t2 − t1 )

ln(3/2)
Δt =
λ

Q7

A = λN

λ =
ln 2

t1/2
=
3×24×60×60
ln 2
sec
−1
= 2.67 × 10
−6
sec
−1

N = Number of atoms in 2mg Au

−3

=
2×10

198
× 6 × 10
23
= 6.06 × 10
15

13 12
A = λN = 1.618 × 10 = 16.18 × 10 dps

Q8

Δm = (ZmP + (A − Z)mn ) − MM e

= (13 × 1.00726 + 14 × 1.00866) − 27.18846

= 27.21562 − 27.18846

= 0.02716u

−3
E = 27.16x × 10  J

Q9

A = A0 e
−λ.t

A0

8
= A0 e
−λt
⇒ λt = ln 8

λt = 3 ln 2

years
ln 2 t 30
= = = 10
λ 3 3

Q10

  #MathBoleTohMathonGo
www.mathongo.com
Nuclear
  Physics JEE Main 2021 (July) Chapter-wise Questions
Hints and Solutions MathonGo

N = N0 e
−λt

 Nd = N0 − N

Nd = N0 (1 − e
−λt
)

Nd

N0
= f = 1 − e
−λt

df −λt
= λe
dt

Q11

Tm = 30 ms

C = 200μF

−t/RC 1
q Q e Q {(λ− )

N
=
0

−λt
=
 N0
0
e
RC

N0 e

Since q/N is constant hence

λ =
1

RC

−3
1 Tm 30×10
R = = = −6
= 150Ω
λC C 200×10

Q12

(A) True, atom of each element emits characteristic

spectrum.

(B) True, according to Bohr's postulates

mvr =
nh


and hence electron resides into

orbits of specific radius called stationary orbits.

(C) False, density of nucleus is constant

(D) False, A free neutron is unstable decays into

proton and electron and antineutrino.

(E) True unstable nucleus show radioactivity.

#MathBoleTohMathonGo
www.mathongo.com
Oscillations
  JEE Main 2021 (July) Chapter-wise Questions
Questions with Answer Keys MathonGo

Q1 (20 July 2021 Shift 2)

A particle is making simple harmonic motion along the X-axis. If at a distances x and x from the mean
1 2

position the velocities of the particle are v and v respectively. The time period of its oscillation is given as:
1 2

2 2
x +x
(1) T = 2π√
2 1

2 2
v −v
1 2

2 2
x +x

(2) T = 2π√
2 1

2 2
v +v
1 2

2 2
x −x

(3) T = 2π√
2 1

2 2
v +v
1 2

2 2
x −x
(4) T = 2π√
2 1

2 2
v −v
1 2

Q2 (22 July 2021 Shift 1)

T0 is the time period of a simple pendulum at a

place. If the length of the pendulum is reduced to

16
1
times of its initial value, the modified time

period is:

(1) T 0

(2) 8πT 0

(3) 4 T 0

(4) 1

4
 T0

Q3 (25 July 2021 Shift 1)

In the reported figure, two bodies A and B of masses 200 g and 800 g are attached with the system of springs.
Springs are kept in a stretched position with some extension when the system is released. The horizontal
surface is assumed to be frictionless. The angular frequency will be rad/s when k = 20 N/m

Q4 (25 July 2021 Shift 1)

  #MathBoleTohMathonGo
www.mathongo.com
Oscillations
  JEE Main 2021 (July) Chapter-wise Questions
Questions with Answer Keys MathonGo

A pendulum bob has a speed of 3 m/s at its lowest position. The pendulum is 50 cm long. The speed of bob,
when the length makes an angle of 60 to the vertical will be (g = 10 m/s )

∘ 2

m/s

Q5 (25 July 2021 Shift 2)

In a simple harmonic oscillation, what fraction of total mechanical energy is in the form of kinetic energy,

when the particle is midway between mean and extreme position.

(1) 1

(2) 3

(3)
1

(4)
1

Q6 (27 July 2021 Shift 1)

A particle starts executing simple harmonic motion (SHM) of amplitude 'a' and total energy E. At any instant,
its kinetic energy is then its displacement 'y' is given by:
3E

(1) y = a
(2) y = a

√2

a √3
(3) y = 2

(4) y = a

Q7 (27 July 2021 Shift 1)

Two identical tennis balls each having mass 'm' and charge ' q ' are suspended from a fixed point by threads of
length ′
r. What is the equilibrium separation when each thread makes a small angle ′
θ ' with the vertical ?
1
2
q l
(1) x = (
2

)
2πε0 mg

1
2
q l
(2) x = (
3

)
2πε0 mg

1
2 2
q l 3

(3) x = ( 2πε0  m  g
2
)

  #MathBoleTohMathonGo
www.mathongo.com
Oscillations
  JEE Main 2021 (July) Chapter-wise Questions
Questions with Answer Keys MathonGo
1
2 2
q l 3

(4) x = ( 2πε0  m  g
2 2
)

Q8 (27 July 2021 Shift 2)

An object of mass 0.5 kg is executing simple harmonic motion. It amplitude is 5 cm and time period (T) is
0.2 s . What will be the potential energy of the object at an instant t = T

4
 s starting from mean position.
Assume that the initial phase of the oscillation is zero.

(1) 0.62 J
(2) 6.2 × 10 −3
 J

(3) 1.2 × 10 3
 J

(4) 6.2 × 10 3
 J

Q9 (27 July 2021 Shift 2)

A particle executes simple harmonic motıon represented by displacement function as

x(t) = A sin(ωt + ϕ)

If the position and velocity of the particle at t = 0 s are 2 cm and 2ωcms −1
respectively, then its amplitude is
x√2 cm where the value of x is

  #MathBoleTohMathonGo
www.mathongo.com
Oscillations
  JEE Main 2021 (July) Chapter-wise Questions
Questions with Answer Keys MathonGo

Answer Key

Q1 (4) Q2 (4) Q3 (10) Q4 (2)

Q5 (2) Q6 (4) Q7 (2) Q8 (1)

Q9 (2)

#MathBoleTohMathonGo
www.mathongo.com
Oscillations
  JEE Main 2021 (July) Chapter-wise Questions
Hints and Solutions MathonGo

Q1

v
2
= ω
2
(A
2
− x )
2

2 2
v v

A
2
= x
2
1
+
ω
1

2
= x
2
2
+
ω
2

2 2
v −v
ω
2
=
2

2
1

x −x
1 2

2 2
x −x
1 2
T = 2π√ 2 2
v −v
2 1

Q2

T0 = 2π√

ℓ/16
New time period T == 2π√  g
=

4

T0
T =
4

Q3
keq
ω = √
μ

μ = reduced mass

springs are in series connection

keq =
k1 k2

k1 +k2

keq  =
k×4k

5k
=
4k

keq  =
4×20

5
 N/m = 16 N/m

m1  m2
μ =
 m1 +m2
=
0.2×0.8

0.2+0.8
= 0.16 kg

16
ω = √ = √100 = 10
0.16

Q4

  #MathBoleTohMathonGo
www.mathongo.com
Oscillations
  JEE Main 2021 (July) Chapter-wise Questions
Hints and Solutions MathonGo

Applying work energy theorem :

wg + wT = ΔK

− mgl(1 − cos 60 ) =
∘ 1

2
mv
2

1

2
mu
2

v
2
= u
2
− 2gl (1 − cos 60 )

2 1
v = 9 − 2 × 10 × 0.5 ( )
2

v
2
= 4

v = 2 m/s

Q5

K =
1

2

2
(A
2
− x )
2

=
1

2
 mω
2
( A
2

A

4
)

1 2 3 A
=  mω ( )
2 4

3 1 2 2
K = ( mω A )
4 2

Q6

1 2
E = Ka
2

3E

4
=
1

2
 K (a
2
− y )
2

4
×
1

2
Ka
2
=
1

2
 K (a
2
− y )
2

y
2
= a
2

3a

a
y =
2

Q7

T cos θ = mg

  #MathBoleTohMathonGo
www.mathongo.com
Oscillations
  JEE Main 2021 (July) Chapter-wise Questions
Hints and Solutions MathonGo
2
kq
T sin θ =
x
2

2
kq
tan θ = 2
x mg

as tan θ ≈ sin θ ≈ x

2 L

2
Kq

x
= 2
2 L x mg

2 1/3
q  L
x = ( )
2πε0 mg

Q8

m
T = 2π√
k

0.5
0.2 = 2π√
k
2
k = 50π

≈500

x = A sin(ωt + ϕ)

ωT
= 5 cm sin( + 0)
4
π
= 5 cm sin( )
2

= 5 cm

1 2
PE = kx
2
2
1 5
= (500)( )
2 100

=0.6255

Q9

x(t) = A sin(ωt + ϕ)

v(t) = Aω cos(ωt + ϕ)

2 = A sin ϕ

2ω = Aω cos ϕ

From (1) and (2) tan ϕ = 1

ϕ = 45

Putting value of ϕ in equation (1) 2 = A {

1
}
√2

  #MathBoleTohMathonGo
www.mathongo.com
Oscillations
  JEE Main 2021 (July) Chapter-wise Questions
Hints and Solutions MathonGo

A = 2 √2

x = 2

#MathBoleTohMathonGo
www.mathongo.com
Ray
  Optics JEE Main 2021 (July) Chapter-wise Questions
Questions with Answer Keys MathonGo

Q1 (20 July 2021 Shift 1)

Region I and II are separated by a spherical surface of radius 25 cm. An object is kept in region I at a distance
of 40 cm from the surface. The distance of the image from the surface is :

(1) 55.44 cm

(2) 9.52 cm
(3) 18.23 cm
(4) 37.58 cm

Q2 (20 July 2021 Shift 1)

An object viewed from a near point distance of 25 cm, using a microscopic lens with magnification '6', gives

an unresolved image. A resolved image is observed at infinite distance with a total magnification double the
earlier using an eyepiece along with the given lens and a tube of length 0.6 m, if the focal length of the

eyepiece is equal to ___ cm.

Q3 (22 July 2021 Shift 1)

A ray of light passes from a denser medium to a

rarer medium at an angle of incidence i. The reflected and refracted rays make an angle of 90 with each other.

The angle of reflection and refraction are respectively r and r . The critical

angle is given by :

  #MathBoleTohMathonGo
www.mathongo.com
Ray
  Optics JEE Main 2021 (July) Chapter-wise Questions
Questions with Answer Keys MathonGo

(1) sin −1
(cotr)

(2) tan −1
(sin i)

(3) sin −1
(tanr')

(4) sin −1
(tanr)

Q4 (22 July 2021 Shift 1)

A ray of light passing through a prism (μ = √3)

suffers minimum deviation. It is found that the

angle of incidence is double the angle of refraction

within the prism. Then, the angle of prism is ____ (in degrees)

Q5 (25 July 2021 Shift 1)

A ray of laser of a wavelength 630 nm is incident

at an angle of 30 at the diamond-air interface. It is


going from diamond to air. The refractive index of

diamond is 2.42 and that of air is 1 . Choose the

correct option.

(1) angle of refraction is 24.41 ∘

(2) angle of refraction is 30 ∘

(3) refraction is not possible

(4) angle of refraction is 53.4 ∘

Q6 (25 July 2021 Shift 2)

A prism of refractive index μ and angle of prism A is placed in the position of minimum angle of deviation. If
minimum angle of deviation is also A, then in terms of refractive index
μ
(1) 2 cos −1
(
2
)

μ
(2) sin −1
(
2
)

  #MathBoleTohMathonGo
www.mathongo.com
Ray
  Optics JEE Main 2021 (July) Chapter-wise Questions
Questions with Answer Keys MathonGo

μ−1
(3) sin −1
(√
2
)

μ
(4) cos −1
(
2
)

Q7 (25 July 2021 Shift 2)

A ray of light entering from air into a denser medium of refractive index , as shown in figure. The light ray
4

suffers total internal reflection at the adjacent surface as shown. The maximum value of angle θ should be
equal to :

√7
(1) sin −1

√5
(2) sin −1

√7
(3) sin −1

√5
(4) sin −1

Q8 (27 July 2021 Shift 1)

A prism of refractive index n and another prism of refractive index n are stuck together (as shown in the
1 2

figure). n and n depend on λ, the wavelength of light, according to the relation

1 2

−14 −14
10.8 × 10 1.8 × 10
n1 = 1.2 +  and n2 = 1.45 +
2 2
λ λ

The wavelength for which rays incident at any angle on the interface BC pass through without bending at that

  #MathBoleTohMathonGo
www.mathongo.com
Ray
  Optics JEE Main 2021 (July) Chapter-wise Questions
Questions with Answer Keys MathonGo

interface will be ___ nm

Q9 (27 July 2021 Shift 2)

The expected graphical representation of the variation of angle of deviation ' δ ' with angle of incidence 'i' in a

prism is :

(1)

(2)

(3)

(4)

  #MathBoleTohMathonGo
www.mathongo.com
Ray
  Optics JEE Main 2021 (July) Chapter-wise Questions
Questions with Answer Keys MathonGo

Answer Key

Q1 (4) Q2 (25) Q3 (4) Q4 (60)

Q5 (3) Q6 (1) Q7 (1) Q8 (600)

Q9 (2)

#MathBoleTohMathonGo
www.mathongo.com
Ray
  Optics JEE Main 2021 (July) Chapter-wise Questions
Hints and Solutions MathonGo

Q1
μ2 μ1 μ2 −μ1

v

u
=
R

1.4−1.25

1.4 1.25
− =
v −40 −25

1.4

 V
= −
0.15

25

1.25

40

v = −37.58 cm

Hence option (4)

Q2

For simple microscope,

m = 1 +
D

f0

6 = 1 +
D

f0

5 =
25

f0

f0 = 5 cm

For compound microscope,

m =
ℓ⋅D

f0 ⋅fe

12 =
60×25

5⋅fe

fe = 25 cm

Q3


r + r + 90

= 180

⇒ r

= 90 − r = 90 − i

n1 sin i = n2 sin r

= n2 sin(90 − i)

n2
n1 sin i = n2 cos i ⇒ tan i =
n1

n2
Now sin C = n1
= tan i

−1 −1
⇒ C = sin (tan i) = sin (tan r)

Q4

At minimum deviation r 1 = r2 =
A

Also given i = 2r 1 = A

  #MathBoleTohMathonGo
www.mathongo.com
Ray
  Optics JEE Main 2021 (July) Chapter-wise Questions
Hints and Solutions MathonGo

Now 1. sin i = √3 sin r


1

1 sin A = √3 sin
A

⇒ 2 sin
A

2
cos
A

2
= √3 sin
A

√3
⇒ cos
A

2
=
2

A

2
= 30


⇒ A = 60

Q5

sin θC =
μ
1
=
2μ2
1
< sin θC

sin θ > sin θC

θ > θc

Total internal reflection will happen

Q6
A+δ
min
sin( )

μ =
A
sin( )
2

A+A
sin( )

μ =
A
sin( )
2

μ =
sin A

A
= 2 cos
A

sin
2

μ
−1
A = 2 cos ( )
2

Q7

At maximum angle θ ray at point B goes in gazing emergence, at all less values of θ, TIR occurs. At point B

  #MathBoleTohMathonGo
www.mathongo.com
Ray
  Optics JEE Main 2021 (July) Chapter-wise Questions
Hints and Solutions MathonGo

3
× sin θ
′′
= 1 × sin 90

θ
′′
= sin
−1
(
3

4
)

θ

= (
π

2
− θ )
′′

At point A 1 × sin θ =

4 ′
× sin θ
3

sin θ =
4

3
× sin(
π

2
− θ )
′′

√7
sin θ =
4

3
cos[cos
−1

4
]

√7

4
sin θ = ×
3 4

√7
−1
θ = sin ( )
3

Q8

For no bending, n 1
= n2

−14 −4

10.8×10 1.8×10
1.2 + = 1.45 +
2 2
λ λ

On solving,

9 × 10
−14
= 25λ
2

λ = 6 × 10
−7

λ = 600 nm

Q9

Standard graph between angle of deviation and

incident angle.

#MathBoleTohMathonGo
www.mathongo.com
Rotational
  Motion JEE Main 2021 (July) Chapter-wise Questions
Questions with Answer Keys MathonGo

Q1 (20 July 2021 Shift 1)

A circular disc reaches from top to bottom of an inclined plane of length 'L'. When it slips down the plane, it takes time 't . ′
1

t2
When it rolls down the plane, it takes time t . The value of is √ The value of x will be ___
3
2 .
t1 x

Q2 (20 July 2021 Shift 2)

A body rolls down an inclined plane without slipping. The kinetic energy of rotation is 50% of its translational kinetic

energy. The body is :

(1) Solid sphere

(2) Solid cylinder

(3) Hollow cylinder


(4) Ring

Q3 (20 July 2021 Shift 2)

Two bodies, a ring and a solid cylinder of same material are rolling down without slipping an

inclined plane. The radii of the bodies are same. The ratio of velocity of the centre of mass at the bottom of the inclined
√x
plane of the ring to that of the cylinder is 2
. Then, the value of x is _____.

Q4 (20 July 2021 Shift 2)

A body rotating with an angular speed of 600 rpm is uniformly accelerated to 1800rpm in 10sec. The number of rotations

made in the process is

Q5 (22 July 2021 Shift 1)

Consider a situation in which a ring, a solid cylinder and a solid sphere roll down on the same inclined plane without

slipping. Assume that they start rolling from rest and having identical diameter. The correct statement for this situation is:-

(1) The sphere has the greatest and the ring has the least velocity of the centre of mass at the bottom of the inclined plane.
(2) The ring has the greatest and the cylinder has the least velocity of the centre of mass at the bottom of the inclined plane.

(3) All of them will have same velocity.

(4) The cylinder has the greatest and the sphere has the least velocity of the centre of mass at the bottom of the inclined

plane.

Q6 (22 July 2021 Shift 1)

  #MathBoleTohMathonGo
www.mathongo.com
Rotational
  Motion JEE Main 2021 (July) Chapter-wise Questions
Questions with Answer Keys MathonGo

6. The centre of a wheel rolling on a plane surface moves with a speed v . A particle on the rim of the wheel at the same
0

level as the centre will be moving at a speed √xv . Then the value of x is ____.
0

Q7 (25 July 2021 Shift 1)

Given below are two statements: one is labelled as Assertion A and the other is labelled as Reason R. Assertion A :

Moment of inertia of a circular disc of mass 'M' and radius 'R' about X, Y axes (passing through its plane) and Z-axis which

is perpendicular to its plane were found to be I x, Iy and I respectively. The respective radii of gyration about all the three
z

axes will be the same.

Reason R : A rigid body making rotational motion has fixed mass and shape. In the light of the above statements, choose

the most appropriate answer from the options given below:

(1) Both A and R are correct but R is NOT the correct explanation of A.

(2) A is not correct but R is correct.

(3) A is correct but R is not correct.

(4) Both A and R are correct and R is the correct explanation of A

Q8 (25 July 2021 Shift 1)

A particle of mass ' m ' is moving in time ' t on a trajectory given by

→ 2
r = 10αt ^i + 5β(t − 5)^
j

Where α and β are dimensional constants.

The angular momentum of the particle becomes the same as it was for t = 0 at time t =

seconds.

Q9 (25 July 2021 Shift 2)

A solid disc of radius 20 cm and mass 10 kg is rotating with an angular velocity of 600rpm, about an axis normal to its

circular plane and passing through its centre of mass. The retarding torque required to bring the disc at rest in 10 s is
−1
π × 10 Nm

Q10 (27 July 2021 Shift 1)

  #MathBoleTohMathonGo
www.mathongo.com
Rotational
  Motion JEE Main 2021 (July) Chapter-wise Questions
Questions with Answer Keys MathonGo

List-I List-II

2
(a) MI of the rod (length L, Mass M, about an axis⊥ to the rod passing through the midpoint) (i)8ML /3

2
(b) MI of the rod (length L, Mass 2M, about an axis ⊥ to the rod passing through one of its end) (ii)ML /3

2
(c) MI of the rod (length 2 L, Mass M, about an axis ⊥ to the rod passing through its midpoint) (iii)ML /12

2
(d) MI of the rod (Length 2L, Mass 2M, about an axis⊥ to the rod passing through one of its end) (iv)2ML /3

Choose the correct answer from the options given below :

(1) (a) − (ii), (b) − (iii), (c) − (i), (d) − (iv)

(2) (a) − (ii), (b) − (i), (c) − (iii), (d) − (iv)

(3) (a) − (iii), (b) − (iv), (c) − (ii), (d) − (i)

(4) (a)-(iii), (b)-(iv), (c)- (i), (d)-(ii)

Q11 (27 July 2021 Shift 1)

The figure shows two solid discs with radius R and r respectively. If mass per unit area is same for both, what is the ratio of

MI of bigger disc around axis AB (Which is ⊥ to the plane of the disc and passing through its centre) of MI of smaller disc

around one of its diameters lying on its plane?

Given 'M' is the mass of the larger disc. (MI stands for moment of inertia)

(1) R 2
: r
2

(2) 2r 4
: R
4

(3) 2R 2
: r
2

(4) 2R 4
: r
4

Q12 (27 July 2021 Shift 2)

In the given figure, two wheels P and Q are connected by a belt B. The radius of P is three

times as that of Q. In case of same rotational

  #MathBoleTohMathonGo
www.mathongo.com
Rotational
  Motion JEE Main 2021 (July) Chapter-wise Questions
Questions with Answer Keys MathonGo
I1
kinetic energy, the ratio of rotational inertias ( I2
)

will be x : 1. The value of x will be

  #MathBoleTohMathonGo
www.mathongo.com
Rotational
  Motion JEE Main 2021 (July) Chapter-wise Questions
Questions with Answer Keys MathonGo

Answer Key

Q1 (2) Q2 (2) Q3 (3) Q4 (200)

Q5 (1) Q6 (2) Q7 (2) Q8 (10)

Q9 (4) Q10 (3) Q11 (4) Q12 (9)

#MathBoleTohMathonGo
www.mathongo.com
Rotational
  Motion JEE Main 2021 (July) Chapter-wise Questions
Hints and Solutions MathonGo

Q1

If disk slips on inclined plane, then it's acceleration

a1 = gsin θ

1 2
L = a1 t
2 1

⇒ t1 = √
2 L

a1
...(i)

If disk rolls on inclined plane, its acceleration,

g sin θ
a2 = I

1+
2
mR

g sin θ
a2 =
mR2

1+
2mR2

a2 =
2

3
 g sin θ

Now L = 1

2
a2 ⋅ t
2
2

⇒ t2 = √
2 L

a2
...(ii)

t2 a1
Now

3
= √ = √
t1 a2 2

⇒ x = 2

Q2

1 2 1 1 2
Iω = × mv
2 2 2

1 2
I =  mR
2

Body is solid cylinder

Q3

  #MathBoleTohMathonGo
www.mathongo.com
Rotational
  Motion JEE Main 2021 (July) Chapter-wise Questions
Hints and Solutions MathonGo

I in both cases is about point of contact

Ring

mgh =
1

2

2

2
v

1 2 R
mgh = (2mR ) 2
2 R

vR = √gh

Solid cylinder

1 2
mgh = Iω
2
2
v

1 3 2 C
mgh = ( mR ) 2
2 2 R

4gh
vC = √
3

vR √3
=
vC 2

Q4

ωf = ω0 + αt

α = 1200 × 6

1 2
θ = ω0 t + αt
2

10

1 1
= 600 × + × 1200 × 6 ×
60 2 36

θ = 200

Q5

g sin θ
a = I
1+
mR2

Iring  > Isolid cylinder  > Isolid sphere 

⇒ aring  < asolid cylinder  < asolid sphere 

⇒ vring  < vsolid cylinder  < vsolid sphere 

Q6

  #MathBoleTohMathonGo
www.mathongo.com
Rotational
  Motion JEE Main 2021 (July) Chapter-wise Questions
Hints and Solutions MathonGo

For no slipping v 0 = ωR

Now v A
= vB = √v
2
0
+ (ωR)
2

= √2v0

⇒ x = 2

Q7

Iz = Ix + Iy (using perpendicular axis theorem) &I = mk 2


( K : radius of gyration )

so mK 2
z
2
= mKx + mKy
2

2
Kz = Kx + Ky
2 2

so radius of gyration about axes x, y&z won't be

same hense asseration A is not correct reason R is

correct statement (property of a rigid body)

Q8
→ 2
r = 10αt ^i + 5β(t − 5)^
j


v = 20α + ^
i + 5β^
j

→ → →
L = m( r × v )

2
= m [10αt ^i + 5β(t − 5)^
j ] × [20αt^
i + 5β^
j]


2^ 2 ^
L = m [50αβt k − 100αβ (t − 5t) k]

→ →
At t = 0, L = 0

50αβt
2
− 100αβ (t
2
− 5t) = 0

  #MathBoleTohMathonGo
www.mathongo.com
Rotational
  Motion JEE Main 2021 (July) Chapter-wise Questions
Hints and Solutions MathonGo

t − 2(t − 5) = 0

t = 10sec

Q9
I(ωf −ωi )

ΔL
τ = =
Δt Δt
2
mR
×[0−ω]

τ =
2

Δt

2
−2
10×(20×10 )

600×π
= ×
2 30×10

−2
= 0.4π = 4π × 10

Q10

Q11
1 2
MR

Ratio of moment of inertia = 2

1 2

mr
4

2 2 4
2σπR R 2R
= 2 2
= 4
σπr r r

Q12

  #MathBoleTohMathonGo
www.mathongo.com
Rotational
  Motion JEE Main 2021 (July) Chapter-wise Questions
Hints and Solutions MathonGo

1 2 1 2
I1 (ω1 ) = I2 (ω2 )
2 2

2 2

I1 (
v

3R
) = I2 (
v

R
)

I1 9
=
I2 1

#MathBoleTohMathonGo
www.mathongo.com
Semiconductors
  JEE Main 2021 (July) Chapter-wise Questions
Questions with Answer Keys MathonGo

Q1 (20 July 2021 Shift 1)

For the circuit shown below, calculate the value of I : z

(1) 25 mA

(2) 0.15 A
(3) 0.1 A
(4) 0.05 A

Q2 (20 July 2021 Shift 2)

A zener diode having zener voltage 8 V and power dissipation rating of 0.5 W is connected across a potential

divider arranged with maximum potential drop across zener diode is as shown in the diagram. The value of
protective resistance R is … … … Ω.

Q3 (20 July 2021 Shift 2)

For the forward biased diode characteristics shown

in the figure, the dynamic resistance at I D = 3 mA

  #MathBoleTohMathonGo
www.mathongo.com
Semiconductors
  JEE Main 2021 (July) Chapter-wise Questions
Questions with Answer Keys MathonGo

will be _____ Ω

Q4 (22 July 2021 Shift 1)

Consider a situation in which reverse biased current of a particular P-N junction increases when it is exposed

to a light of wavelength ≤ 621 nm. During this process, enhancement in carrier concentration takes place due
to generation of hole-electron pairs. The value of band gap is nearly.

(1) 2eV
(2) 4eV
(3) 1eV
(4) 0.5eV

Q5 (22 July 2021 Shift 1)

In a given circuit diagram, a 5 V zener diode along with a series resistance is connected across a 50 V power

supply. The minimum value of the resistance required, if the maximum zener current is 90 mA will be ____Ω

  #MathBoleTohMathonGo
www.mathongo.com
Semiconductors
  JEE Main 2021 (July) Chapter-wise Questions
Questions with Answer Keys MathonGo

Q6 (25 July 2021 Shift 1)

Identify the logic operation carried out.

(1) OR
(2) AND
(3) NOR
(4) NAND

Q7 (25 July 2021 Shift 2)

In a semiconductor, the number density of intrinsic charge carriers at 27 ∘


C is 1.5 × 10 16
/m
3
. If the

semiconductor is doped with impurity atom, the hole density increases to 4.5 × 10 22
/m
3
. The

electron density in the doped semiconductor is − × 10 9


/m
3

Q8 (27 July 2021 Shift 1)

A transistor is connected in common emitter circuit configuration, the collector supply voltage is 10 V and the
voltage drop across a resistor of 1000Ω in the collector circuit is 0.6 V. If the current gain factor (β) is 24 ,
then the base current is ___ μA. (Round off to the Nearest Integer)

Q9 (27 July 2021 Shift 2)

Find the truth table for the function Y of A and B represented in the following figure.

(1)

  #MathBoleTohMathonGo
www.mathongo.com
Semiconductors
  JEE Main 2021 (July) Chapter-wise Questions
Questions with Answer Keys MathonGo

A B Y

0 0 0

0 1 1

1 0 0

1 1 0

(2)

A B Y

0 0 1

0 1 0

1 0 1

1 1 1

(3)

$A$ $B$ $Y$

0 0 0

0 1 0

1 0 0

1 1 1

(4)

A B Y

0 0 0

0 1 1

1 0 1

1 1 1

  #MathBoleTohMathonGo
www.mathongo.com
Semiconductors
  JEE Main 2021 (July) Chapter-wise Questions
Questions with Answer Keys MathonGo

Answer Key

Q1 (1) Q2 (192) Q3 (25) Q4 (1)

Q5 (500) Q6 (2) Q7 (5) Q8 (25)

Q9 (2)

#MathBoleTohMathonGo
www.mathongo.com
Semiconductors
  JEE Main 2021 (July) Chapter-wise Questions
Hints and Solutions MathonGo

Q1

50
I = = 50 mA
1000

R = 1000Ω

I =
50

2000
= 25 mA

IZ = I1000 − I2000

= 50 − 25 = 25 mA

Q2

P = Vi

0.5 = 8i

1
i =  A
16

E = 20 = 8 + iRP

RP = 12 × 16 = 192Ω

Q3

dV 1 1
Rd = = di
=
di 5−1×10−3

dv 0.75−0.65

100
= 25Ω
4

Q4

Band gap = hc

λ0

λ0 ; threshold wavelength

1242ev−nm
Band gap = 621 nm
= 2eV

Q5

  #MathBoleTohMathonGo
www.mathongo.com
Semiconductors
  JEE Main 2021 (July) Chapter-wise Questions
Hints and Solutions MathonGo

Voltage across R L
= 5 V

⇒ i2 =
5

RL

Also voltage across R = 50 − 5 = 45 volt

By v = iR ⇒ R =

v 45
=
i ii +i2

45
R = 5
90 mA+
R
L

Current in zener diode is maximum when R L → ∞

(i2 → 0 and ii = i)

45
So R = 90 mA
= 500Ω

Q6

Q7

ne nh = n
2

16 2
2
n (1.5×10 )

ne =
nh
i
=
4.5×10
22

32

=
1.5×1.5×10

22

4.5×10

9 3
5 × 10 /m

Q8

  #MathBoleTohMathonGo
www.mathongo.com
Semiconductors
  JEE Main 2021 (July) Chapter-wise Questions
Hints and Solutions MathonGo
IC
β =
IB
= 24; RC = 1000

ΔV = 0.6

IC =
0.6

1000

IC = 6 × 10
−4

−4
IC 6×10
IB = = = 25μA
β 24

Q9

¯
¯¯¯
Y = A ⋅ B + B

A B Y

0 0 1

0 1 0

1 0 1

1 1 1

#MathBoleTohMathonGo
www.mathongo.com
Thermal
  Properties of Matter JEE Main 2021 (July) Chapter-wise Questions
Questions with Answer Keys MathonGo

Q1 (22 July 2021 Shift 1)

In 5 minutes, a body cools from 75 ∘


C to 65 ∘
C at room temperature of 25 ∘
C . The temperature of body at the

end of next 5 minutes is ____.

Q2 (25 July 2021 Shift 1)

Two different metal bodies A and B of equal mass are heated at a uniform rate under similar

conditions. The variation of temperature of the

bodies is graphically represented as shown in the figure. The ratio of specific heat capacities is :

(1) 8

(2) 3

(3) 3

(4) 4

Q3 (27 July 2021 Shift 1)

A body takes 4 min. to cool from 61 ∘


C to 59 ∘
C . If the temperature of the surroundings is 30 ∘
C , the time taken

by the body to cool from 51 ∘


C to 49 ∘
C is:

(1) 4 min.

(2) 3 min.
(3) 8 min.
(4) 6 min.

  #MathBoleTohMathonGo
www.mathongo.com
Thermal
  Properties of Matter JEE Main 2021 (July) Chapter-wise Questions
Questions with Answer Keys MathonGo

Answer Key

Q1 (57) Q2 (2) Q3 (4)

#MathBoleTohMathonGo
www.mathongo.com
Thermal
  Properties of Matter JEE Main 2021 (July) Chapter-wise Questions
Hints and Solutions MathonGo

Q1

By newton's law of cooling (with approximation)

ΔT

Δt
= −C (Tavg  − Ts )


st  −10 C
1
5 min
= −C (70 C − 25 C)
∘ ∘

⇒ C =
45
2
 min
−1

nd  T−65 T+65 T+15


2
5 min
= −C (
2
− 25) = − (
2

45
)(
2
)

⇒ 9( T − 65) = −(T + 15)

⇒ 10 T = 570

⇒ T = 57 C

Alternate Solution :

Newton's law of cooling (without approximation)

TP − TS = (Ti − TS ) e
−Ct

st  −5C −5C 4


1 65 − 25 = (75 − 25)e ⇒ e =
5

2
nd 
T − 25 = (65 − 25)e
−5C
= 40 ×
4

5
= 32


 T = 57 C

Q2
ΔQ ΔQ
(
Δt
) = (
Δt
)

A B

ΔT ΔT
mSA ( ) = mSB ( )
Δt Δt
A B

Q3

ΔT

Δt
= K (Tt − Ts ) Tt = average temp.

TS = surrounding temp

61−59 61+59

4
= K(
2
− 30) ...(1)

51−49 51+49

t
= K (
2
− 30) ...(2)

Divide (1) & (2) t

4
=
60−30

50−30
=
30

20

so, t = 6 minutes

 
#MathBoleTohMathonGo
www.mathongo.com
Thermodynamics
  JEE Main 2021 (July) Chapter-wise Questions
Questions with Answer Keys MathonGo

Q1 (20 July 2021 Shift 1)

The amount of heat needed to raise the temperature of 4 moles of a rigid diatomic gas from 0 ∘
C to 50 ∘
C when
no work is done is ___ (R is the universal gas constant)

(1) 250R

(2) 750R

(3) 175R

(4) 500R

Q2 (20 July 2021 Shift 1)

The entropy of any system is given by

μkR
2
S = α β ln[ + 3]
2

where α and β are the constants. μ, J, k and R are

no. of moles, mechanical equivalent of heat, Boltzmann constant and gas constant respectively.

dQ
[ Take S = ]
T

Choose the incorrect option from the following:

(1) α and J have the same dimensions.


(2) S, β, k and μR have the same dimensions.

(3) S and α have different dimensions.


(4) α and k have the same dimensions.

Q3 (20 July 2021 Shift 1)

  #MathBoleTohMathonGo
www.mathongo.com
Thermodynamics
  JEE Main 2021 (July) Chapter-wise Questions
Questions with Answer Keys MathonGo

In the reported figure, heat energy absorbed by a system in going through a cyclic process is ___ πJ

Q4 (20 July 2021 Shift 2)

Which of the following graphs represent the behavior of an ideal gas ? Symbols have their usual meaning.

(1)

(2)

(3)

  #MathBoleTohMathonGo
www.mathongo.com
Thermodynamics
  JEE Main 2021 (July) Chapter-wise Questions
Questions with Answer Keys MathonGo

(4)

Q5 (20 July 2021 Shift 2)

One mole of an ideal gas at 27 ∘


C is taken from A

to B as shown in the given PV indicator diagram.

The work done by the system will be ×10 −1


 J .

[ Given : R = 8.3 J/moleK, ln 2 = 0.6931]

(Round off to the nearest integer)

Q6 (25 July 2021 Shift 1)

A monoatomic ideal gas, initially at temperature T


1

is enclosed in a cylinder fitted with a frictionless

piston. The gas is allowed to expand adiabatically

to a temperature T by releasing the piston

suddenly. If I and I are the lengths of the gas

1 2

  #MathBoleTohMathonGo
www.mathongo.com
Thermodynamics
  JEE Main 2021 (July) Chapter-wise Questions
Questions with Answer Keys MathonGo

column, before and after the expansion

T1
respectively, then the value of  T2
will be
2

l1 3

(1) ( I2
)

(2) (
l2 3

)
l1

(3) l2

l1

(4)
l1

l2

Q7 (25 July 2021 Shift 2)

A heat engine has an efficiency of . When the temperature of sink is reduced by 62 , its efficiency get
1 ∘
C
6

doubled. The temperature of the source is :

(1) 124 ∘
C

(2) 37 ∘
C

(3) 62 ∘
C

(4) 99 ∘
C

Q8 (27 July 2021 Shift 1)

In the reported figure, there is a cyclic process ABCDA on a sample of 1 mol of a diatomic gas. The

temperature of the gas during the process A → B and C → D are T and T 1 2 ( T1 > T2 ) respectively.

Choose the correct option out of the following for work done if processes BC and DA are adiabatic.

  #MathBoleTohMathonGo
www.mathongo.com
Thermodynamics
  JEE Main 2021 (July) Chapter-wise Questions
Questions with Answer Keys MathonGo

(1) W AB
= WDC

(2) W AD
= WBC

(3) W BC
+ WDA > 0

(4) W AB
< WCD

Q9 (27 July 2021 Shift 2)

One mole of an ideal gas is taken through an adiabatic process where the temperature rises from 27 ∘
C to 37 ∘
C

. If the ideal gas is composed of polyatomic molecule that has 4 vibrational modes, which of the following is
true? [R = 8.314 J mol −1
k
−1
]

(1) work done by the gas is close to 332 J

(2) work done on the gas is close to 582 J


(3) work done by the gas is close to 582 J

(4) work done on the gas is close to 332 J

Q10 (27 July 2021 Shift 2)

Two Carnot engines A and B operate in series such that engine A absorbs heat at T and rejects heat to a sink
1

at temperature T. Engine B absorbs half of the heat rejected by Engine A and rejects heat to the sink at T . 3

When workdone in both the cases is equal, to value of T is :

(1) 2

3
 T1 +
3

2
 T3

(2)
1 2
 T1 +  T3
3 3

(3) 3

2
 T1 +
1

3
 T3

(4)
2 1
 T1 +  T3
3 3

  #MathBoleTohMathonGo
www.mathongo.com
Thermodynamics
  JEE Main 2021 (July) Chapter-wise Questions
Questions with Answer Keys MathonGo

Answer Key

Q1 (4) Q2 (4) Q3 (100) Q4 (3)

Q5 (17258) Q6 (2) Q7 (4) Q8 (2)

Q9 (2) Q10 (4)

#MathBoleTohMathonGo
www.mathongo.com
Thermodynamics
  JEE Main 2021 (July) Chapter-wise Questions
Hints and Solutions MathonGo

Q1

ΔQ = ΔU + ΔW

Here ΔW = 0

ΔQ = ΔU = nCV ΔT

5R
ΔQ = 4 × (50) = 500R
2

Hence option (4).

Q2
μKR
2
S = α βℓn ( 2
+ 3)

Q
S =
T
= joule /k

2
[α β] = Joule /k
μKR
PV = nRT [ 2
] = 1

Joule
R =
K

Joule Joule
⇒ R = ,K =
K R

 Joule 
⇒ β = ( )
K

2 Joule
α β = ( )
K

⇒ α = dimensionless

Q3

For complete cyclic process

#MathBoleTohMathonGo
www.mathongo.com
Thermodynamics
  JEE Main 2021 (July) Chapter-wise Questions
Hints and Solutions MathonGo

ΔU = 0

∴ from ΔQ = ΔU + W

= 0 + W

ΔQ = W

= Area

= πr1 ⋅ r2

= π × (10 × 10 ) × (10 × 10
3 −3
)

ΔQ = 100π

∴ Ans. = 100

Q4

PV = nRT

PV ∝ T

Straight line with positive slope (nR)

Q5

Process of isothermal

V2
W = nRTℓn ( )
 V1

= 1 × 8.3 × 300 × ln 2

−1
= 17258 × 10 J

Q6

PV
r
= const. TV [−1
= const

T(ℓ) 3
−1
= const

2/3
T1 ℓ2
= ( )
T2 ℓ1

Q7

#MathBoleTohMathonGo
www.mathongo.com
Thermodynamics
  JEE Main 2021 (July) Chapter-wise Questions
Hints and Solutions MathonGo
TL
η = 1 −
TH
… (i)

(TL −62) TL 62
2η = 1 − = 1 − +
TH TH TH

62 1 62
⇒ η = ⇒ = ⇒ TH = 6 × 62 = 372 K
TH 6 TH

∘ ∘
In C ⇒ 372 − 273 = 99 C

Q8
−nR
Work done in adiabatic process = γ−1
( Tf − Ti )

−nR
∴ WAD = ( T2 − T1 )
γ−1

−nR
and W BC
=
γ−1
( T2 − T1 )

∴ WAD = WBC

Q9

Since, each vibrational mode, corresponds to two degrees of freedom, hence, f = 3 (trans.) +3 (rot.) + 8( vib.
)= 14

2
& γ = 1 +
f

2 8
γ = 1 + =
14 7

nRΔT
W = = −582
γ−1

As W < 0. work is done on the gas.

Q10

Q2 T Q2 T
WA = 1 − = 1 − ⇒ =
Q T1 Q T1
1 1

Q T3 2Q T3
3 3
WB = 1 − = 1 − ⇒ =
(Q /2) T Q T
2 2

#MathBoleTohMathonGo
www.mathongo.com
Thermodynamics
  JEE Main 2021 (July) Chapter-wise Questions
Hints and Solutions MathonGo

Now, W A
= WB

Q2
Q1 − Q2 = − Q3
2

2Q1 2Q
3
⇒ + = 3
Q Q
2 2

2 T1 T3
⇒ + = 3
T T

2 T1 T3
+ = T
3 3

#MathBoleTohMathonGo
www.mathongo.com
Units
  and Dimensions JEE Main 2021 (July) Chapter-wise Questions
Questions with Answer Keys MathonGo

Q1 (20 July 2021 Shift 2)

If time (t), velocity (v), and angular momentum (l) are taken as the fundamental units. Then the

dimension of mass (m) in terms of t, v and l is :

(1) [t −1
v l
1 −2
]

(2) [t 1 2 −1
v l ]

(3) [t −2
v
−1 l
l ]

(4) [t −1
v
−2 1
l ]

Q2 (25 July 2021 Shift 2)

The force is given in terms of time t and

displacement x by the equation

F = A cos Bx + C sin Dt

The dimensional formula of AD

B
is :

(1) [M 0
 L T
−1
]

(2) [ML 2
 T
−3
]

(3) [M 1 1
 L  T
−2
]

(4) [M 2 2
 L  T
−3
]

Q3 (27 July 2021 Shift 2)

Match List I with List II.

List-I List-II

(a) Capacitance, C

(i) M 1
 L  T
1 −3
 A
−1

(b) Permittivity of free space, ε


0

(ii) M −1
 L
−3 4
 T  A
2

(c) Permeability of free space, μ


0

(iii) M −1
 L
−2
 T  A
4 2

  #MathBoleTohMathonGo
www.mathongo.com
Units
  and Dimensions JEE Main 2021 (July) Chapter-wise Questions
Questions with Answer Keys MathonGo

(d) Electric field, E

(iv) M 1 1
 L  T
−2
 A
−2

Choose the correct answer from the options given

below

(1) (a) → (iii), (b) → (ii), (c) → (iv), (d) → (i)

(2) (a) → (iii), (b) → (iv), (c) → (ii), (d) → (i)


(3) (a) → (iv), (b) → (ii), (c) → (iii), (d) → (i)

(4) (a) → (iv), (b) → (iii), (c) → (ii), (d) → (i)

  #MathBoleTohMathonGo
www.mathongo.com
Units
  and Dimensions JEE Main 2021 (July) Chapter-wise Questions
Questions with Answer Keys MathonGo

Answer Key

Q1 (4) Q2 (2) Q3 (1)

#MathBoleTohMathonGo
www.mathongo.com
Units
  and Dimensions JEE Main 2021 (July) Chapter-wise Questions
Hints and Solutions MathonGo

Q1

m ∝ t v ℓ
a b c

b c

m ∝ [T] [LT
a −1
] [ML  T
2 −1
]

1
M L T
0 0
= M L
c b+2c
T
a−b−c

comparing powers c = 1,  b = −2, a = −1

−1 −2 1
m ∝ t V ℓ

Q2

−2
[A] = [MLT ]

[B] = [L
−1
]

[D] = [T
−1
]

AD 2 −3
[ ] = [ML  T ]
B

Q3

q = CV

2
q (A×T)
[C] = [
V
] = 2 −2

ML  T

= M
−1
 L
−2
 T  A
4 2

−2

[E] = [
F

q
] =
MLT

AT

−3 −1
= MLT  A

q q

1 2
F = 2
4π∈o r

−1 −3 4 2
[ϵo ] = M  L  T  A

Speed of light c =

√μ o ∈ ∘

1
μ0 = 2
ϵ0 c

1
[μ0 ] =
2
−1 −3 4 2 −1
[M  L  T  A ][LT ]

1 1 −2 −2
= [M  L  T  A ]

#MathBoleTohMathonGo
www.mathongo.com
Wave
  Optics JEE Main 2021 (July) Chapter-wise Questions
Questions with Answer Keys MathonGo

Q1 (25 July 2021 Shift 1)

In the Young's double slit experiment, the distance between the slits varies in time as

d(t) = d0 + a0 sinot ; where d 0, ω and a are

constants. The difference between the largest fringe width and the smallest fringe width obtained

over time is given as :


2λD(d0 )
(1) 2 2
(d −a )
0 0

(2) 2λDa0

2 2
( d0 −a )
0

(3) λD
2
a0
d
0

(4) λD

d0 +a0

Q2 (27 July 2021 Shift 1)

In Young's double slit experiment, if the source of light changes from orange to blue then:

(1) the central bright fringe will become a dark fringe.


(2) the distance between consecutive fringes will decrease.

(3) the distance between consecutive fringes will increase.

(4) the intensity of the minima will increase.

Q3 (27 July 2021 Shift 2)

The difference in the number of waves when yellow light propagates through air and vacuum columns of the
same thickness is one. The thickness of the air column is mm. [Refractive index of air = 1.0003, wavelength
of yellow light in vacuum = 6000Ã …]

  #MathBoleTohMathonGo
www.mathongo.com
Wave
  Optics JEE Main 2021 (July) Chapter-wise Questions
Questions with Answer Keys MathonGo

Answer Key

Q1 (2) Q2 (2) Q3 (2)

#MathBoleTohMathonGo
www.mathongo.com
Wave
  Optics JEE Main 2021 (July) Chapter-wise Questions
Hints and Solutions MathonGo

Q1

Fringe Width, β = λD

βaax ⇒ dmin and β min ⇒ dmax 

d = d0 + a0 sin ωt

dmax = d0 + a0 and d min = d0 − a0

βmin =
λD

d0 +a0
and ∴ β max =
λD

d0 −a0

Q2

Fringe width = λD/d

as λ decreases, fringe width also decreases

Q3

Thickness t = nλ

So, n λ vac  = (n + 1)λair 

n λ = (n + 1) λ

μair

n =
1

μair −1
=
10

t = nλ

=
10

3
× 6000 Ã …

= 2 mm

#MathBoleTohMathonGo
www.mathongo.com
Waves
  and Sound JEE Main 2021 (July) Chapter-wise Questions
Questions with Answer Keys MathonGo

Q1 (20 July 2021 Shift 1)

The amplitude of wave disturbance propagating in the positive x -direction is given by y = at time
1

2
(1+x)

t = 0 and y = 1

1+(x−2)
2
at t = ls, where x and y are in meres. The shape of wave does not change during the

propagation. The velocity of the wave will be ___ m/s

Q2 (20 July 2021 Shift 1)

The frequency of a car horn encountered a change from 400 Hz to 500 Hz. When the car approaches a vertical

wall. If the speed of sound is 330 m/s. Then the speed of car is ___ km/h.

Q3 (20 July 2021 Shift 2)

With what speed should a galaxy move outward

with respect to earth so that the sodium-D line at wavelength 5890Ã … is observed at 5896Ã … ?

(1) 306 km/sec

(2) 322 km/sec
(3) 296 km/sec

(4) 336 km/sec

  #MathBoleTohMathonGo
www.mathongo.com
Waves
  and Sound JEE Main 2021 (July) Chapter-wise Questions
Questions with Answer Keys MathonGo

Answer Key

Q1 (2) Q2 (132) Q3 (1)

#MathBoleTohMathonGo
www.mathongo.com
Waves
  and Sound JEE Main 2021 (July) Chapter-wise Questions
Hints and Solutions MathonGo

Q1

At t = 0, y =

2
1+x

At time t = t, y = 1+(x−vt)
1

At t = 1, y = 1+(x−v)
1

2
… (i)

At t = 1, y = 1+(x−2)
1

2
… (ii)

Comparing (i) & (ii)

v = 2 m/s

Q2

Wall as an observer

Frequency received by wall

f1 = f0 (
C−V
C
)

Again wall as a source

Frequency received by observer on car

C+V
f2 = f1 (
C
)

C+V
f2 = f0 (
C−V
)

C+V
500 = 400 (
C−V
)

C+V

5
=
4 C−V

C = 9 V

V =
C

9
=
330

9
 m/s

330 18
V = × = 132 km/h
9 5

Q3

  #MathBoleTohMathonGo
www.mathongo.com
Waves
  and Sound JEE Main 2021 (July) Chapter-wise Questions
Hints and Solutions MathonGo

1+β

v
f = f0 √ β =
1−β c

1+β

f0
f

1−β

(1 +
Δf

f0
) = (1 + β)(1 − β)
−1

β is small compared to 1

(1 +
2Δf

f0
) = (1 + 2β)

Δf v
β = =
f0 c

c
v = 6 × = 305.6 km/s
5890

#MathBoleTohMathonGo
www.mathongo.com
Work
  Power Energy JEE Main 2021 (July) Chapter-wise Questions
Questions with Answer Keys MathonGo

Q1 (20 July 2021 Shift 2)

If the Kinetic energy of a moving body becomes four times its initial Kinetic energy, then the percentage

change in its momentum will be :

(1) 100%

(2) 200%

(3) 300%

(4) 400%

Q2 (22 July 2021 Shift 1)

A porter lifts a heavy suitcase of mass 80 kg and at the destination lowers it down by a distance of 80 cm with

a constant velocity. Calculate the workdone by the porter in lowering the suitcase. (take g = 9.8 ms −2
)

(1) −62720.0 J

(2) −627.2 J
(3) +627.2 J

(4) 784.0 J

Q3 (25 July 2021 Shift 2)

A force of F = (5y + 20)^jN acts on a particle. The workdone by this force when the particle is moved from
y = 0 m to y = 10 m is

Q4 (27 July 2021 Shift 2)

Given below is the plot of a potential energy function U(x) for a system, in which a particle is in one
dimensional motion, while a conservative force F(x) acts on it. Suppose that E mech 
= 8 J , the incorrect

  #MathBoleTohMathonGo
www.mathongo.com
Work
  Power Energy JEE Main 2021 (July) Chapter-wise Questions
Questions with Answer Keys MathonGo

statement for this system is :

(1) at x > x 4,  K. E . is constant throughout the region.


(2) at x < x 1,  K. E . is smallest and the particle is moving at the slowest speed.
(3) at x = x 2,  KE. is greatest and the particle is moving at the fastest speed.
(4) at x = x 3,  K. E. = 4 J .

Q5 (27 July 2021 Shift 2)

An automobile of mass ' m ' accelerates starting

from origin and initially at rest, while the engine

supplies constant power P. The position is given as

a function of time by:


1
3
9P
(1) (
2

) t2
8 m

1
2

(2) ( 8P 2

) t3
9 m

1
3
9 m
(3) (
2

) t2
8P

1
3

(4) ( 8P 2

) t2
9 m

  #MathBoleTohMathonGo
www.mathongo.com
Work
  Power Energy JEE Main 2021 (July) Chapter-wise Questions
Questions with Answer Keys MathonGo

Answer Key

Q1 (1) Q2 (2) Q3 (450) Q4 (2)

Q5 (4)

#MathBoleTohMathonGo
www.mathongo.com
Work
  Power Energy JEE Main 2021 (July) Chapter-wise Questions
Hints and Solutions MathonGo

Q1

K2 = 4 K1

2
mv
2
2
= 4
1

2
mv
2
1

v2 = 2v1

P = mv

P2 = mv2 = 2mv1

P1 = mv1

2mv1 −mv1
% change = ΔP

P1
× 100 =
mv1
× 100 = 100%

Q2

WPorter  + Wmg = ΔK. E. = 0

 WPorter  = −Wmg = −mgh

= −80 × 9.8 × .8 = −627.2 J

Q3

F = (5y + 20)^
j

10
ω = ∫ F dy = ∫
0
(5y + 20)dy

2 10
5y
= (
2
+ 20y)

=
5

2
× 100 + 20 × 10

= 250 + 200 = 450 J

Q4

Emech.  = 8 J

 (A) at x > x4 , U =  constant  = 6 J

 K = Emech  − U = 2 J =  constant 

 (B) at x < x1 , U =  constant  = 8 J

 K = Emech.  − U = 8 − 8 = 0 J

  #MathBoleTohMathonGo
www.mathongo.com
Work
  Power Energy JEE Main 2021 (July) Chapter-wise Questions
Hints and Solutions MathonGo

Particle is at rest.

(C) At x = x 2, U = 0 ⇒ Emech.  = K = 8 J

KE is greatest, and particle is moving at fastest speed.

x = x3 ,U = 4 J

 (D) At  U + K = 8 J

 K = 4 J

Q5

P = const.

P = Fv =
mv dv

dx

x v

0 m
P
dx = ∫
0
2
v  dv

3
Px

m
=
v

1/3
3Px
(
m
) = v =
dx

dt

1/3
t x
(
3P

m
) ∫
0
dt = ∫
0
x
−1/3
dx

1/2
8P
3/2
⇒ x = ( ) t
9 m

#MathBoleTohMathonGo
www.mathongo.com

You might also like